You are on page 1of 53

GS SCORE

PRELIMS TEST
SERIES 2017
TEST - 27

Mock 1 Paper 1 and


Paper 2

Offc. No. 6, Ist Floor, Apsara Arcade, Karol Bagh, New Delhi-5, (Karol Bagh Metro Gate No. 5)
9953595114, 9873245114 www.iasscore.in www.facebook.com/iasscore
Paper Code: 1 A

Mock - 1 Paper - 1

E
ANSWERS

1. (d)
Exp: NBAGR is premier institute for
identification, characterization, evaluation,
OR2. (a)
Exp: A genetically modified insect is an insect
that has been genetically modified for
conservation and utilization of livestock various reasons such as agricultural
and poultry genetic resources of the production, oil production and pest control.
SC
country. It is located in Karnal, Haryana.
Scientists have moved on from using bed
It was established by merging National
nets and insecticides to kill malaria-
Bureau of Animal Genetic Resources and
spreading mosquitoes, to genetically modify
National Institute of Animal Genetics in
the mosquitoes by inserting a gene that
September 1995.
leads to the production of male offsprings.
NBAGR Objectives:
GS

Since only females carry the malaria-


• To conduct systematic surveys to causing microorganism, the spread of the
characterise, evaluate and catalogue disease is controlled in the short-term while
farm livestock and poultry genetic eventually the whole population gets wiped
resources and to establish their National out.
Data Base. Scientists injected a gene from a slime mould
• To design methodologies for ex situ into the mosquito which attached itself to
conservation and in situ management the X chromosome during sperm-making
and optimal utilization of farm animal process effectively masking the sperms
genetic resources. leading to production of male offsprings.

• To undertake studies on genetic In response to the Zika virus outbreak,


characterisation using modern biological Brazil's National Biosafety Committee
techniques such as molecular approved the releases of more genetically
cytogenetics, Immunology, DNA modified Aedes aegypti mosquitos
Fingerprinting, RFLP analysis etc. throughout their country.

• To conduct training programmes as 3. (a)


related to evaluation, characterisation Exp: Faster growth may not be always assured
and utilisation of animal genetic in the usage of compost. That's why we use
resources. chemical fertilizers in the fields.

Answers: [1]
4. (a) • Malabar-15: India and US have been
regularly participating. Recently included
Exp: Punjabis are known for celebrating participation by Japanese Maritime Self
Baisakhi. Defence Force (JMSDF).
5. (c) • Varuna-15: Since 1993 Indo-French Naval
Exercise 'VARUNA'.
Exp: Earthquake waves are basically of two types
- body waves (P wave and S wave) and • Simbex-15: Singapore India Maritime
surface waves (love wave and Rayleigh Bilateral Exercise alternately in the Bay of
wave). The P-waves are similar to sound Bengal and in South China Sea.
waves. They travel through gaseous, liquid
and solid materials. P-waves vibrate parallel • Konkan-15: The annual bilateral naval
to the direction of the wave. This exerts exercise KONKAN with the Royal Navy,
pressure on the material in the direction of held in 2015 off the coast of the UK.
the propagation. As a result, it creates • Ausindex-15: Between IN and Royal
density differences in the material leading Australian Navy (RAN).
to stretching and squeezing of the material.

E
• Slinex-15: First time in 2005 and in 2010,
S-waves arrive at the surface with some in 2015 off Sri Lanka.
time lag. These are called secondary waves.
OR
An important fact about S-waves is that • Indra-15: Indo - Russia naval exercise
they can travel only through solid materials. named 'INDRA' is being conducted since
The direction of vibrations of S-waves is 2003.
perpendicular to the wave direction in the
vertical plane. Hence, they create troughs 10. (c)
and crests in the material through which
SC

Exp: Hindustani Sangeet is usually considered


they pass. Surface waves are considered to to be a mixture of traditional Hindu musical
be the most damaging waves. concepts and Persian performance practice.
Shadow zone of P wave is smaller than S Types of Hindustani Music and its meaning
wave shadow zone.
Dhrupad - Effort from vocal chords and
6. (a) lungs.
GS

7. (d) Dhamar - Play of Krishna during holy.


Exp: Rise in imports will increase the supply and Thumri - Romantic religious literature.
hence fulfill the gap of demand, hence
prices will not rise; whereas rise in exports Bhajan - Religious devotional songs.
can cause inflation.
11. (d)
8. (b)
12. (d)
Exp: It seeks to provide connectivity to un-served
Exp: The Ministry of Skill Development and
and under-served airports through revival
Entrepreneurship (SDE) and Wadhwani
of existing air-strips and airports etc.
Operating Foundation (WOF) have signed
Recently, The Delhi HC issued notices to
two Agreements for effectuating an earlier
the Centre, the AAI and DGCA as the
understanding of jointly developing and
Federation of Indian Airlines (FAI) sought
implementing Entrepreneurship and Skill
a stay on Government notification on levy
Development initiatives in the country with
of Rs.7, 500 to Rs.8, 500 per flight to create
the Pradhan Mantri YUVA Scheme being
a regional connectivity fund (RCF).
the first joint effort in this regard. Pradhan
9. (c) Mantri YUVA Scheme launched by the
Ministry during November, 2016 aims at
Exp: Exercises with Foreign Navies: providing online Entrepreneurship

[ 2 ] Answers:
Education to about 15 lakh students across Consequently the relative humidity drops.
the country through 3050 Project Institutes This area is called rain shadow zone
consisting of Institutions of Higher
Learning; Schools; ITIs and 18. (c)
Entrepreneurship Development Centres, Exp: Varying from bank to bank, the micro-
both Government and Private. The online ATM allows for biometric identification (via
Entrepreneurship Modules will be duly Aadhaar), use of debit and RuPay cards,
supplemented by the class room based or the mobile phone number, to access bank
experiential learning activities and
account. The basic transaction types that
practicums which will be facilitated by
are normally supported by a Micro-ATM
specially trained Faculty of the educational
are; deposit, withdrawal, fund transfer and
institutions.
balance enquiry. These ATMs offer an
13. (c) online interoperable, low-cost payments
platform to everyone in the country. These
Exp: A group of farmers form a co-operative
also offer a host of features such as e-KYC
society by pooling in their resources

E
based account opening, account servicing,
voluntarily for more efficient and profitable
farming. Individual farms remain intact remittances, utility payments, account
servicing etc, apart from withdrawal and
and farming is a matter of cooperative
initiative.
Co-operative societies help farmers, to
OR 19.
deposit facility.
(c)
procure all important inputs of farming, sell Exp: NGT is a statutory body under National
the products at the most favourable terms
Green Tribunal Act 2010. The Tribunal is
SC
and help in processing of quality products
not bound by the procedure laid down
at cheaper rates.
under the Code of Civil Procedure, 1908,
Co-operative movement originated over a but shall be guided by principles of natural
century ago and has been successful in justice.
many western European countries like
Denmark, Netherlands, Belgium, Sweden, 20. (d)
GS

Italy etc. In Denmark, the movement has Exp: Godavari is the second largest river in India
been so successful that practically every after Ganga. Penna drains into Bay of
farmer is a member of a co-operative. Bengal. Koyna dam is situated on Koyna
Collective farming or the model of Kolkhoz river , which is a tributary of Krishna river.
was introduced in erstwhile Soviet Union
21. (d)
to improve upon the inefficiency of the
previous methods of agriculture and to Exp: The Reserve Bank of India (RBI) has retained
boost agricultural production for self- the classification of State Bank of India and
sufficiency. ICICI Bank Ltd as Domestic Systemically
14. (c) Important Banks (D-SIBs) in 2016. SIBs are
big banks of the country that are perceived
15. (d) "too big to fail". These are critical for the
16. (b) uninterrupted availability of essential
banking services to the country's real
Exp: Preventing conflict and sustaining peace economy even during crisis.
through decent work
• There are two types of SIBs: Global SIBs;
17. (b) the identification is done by BASEL
Exp: In the leeward side, a decrease in altitude committee on banking supervision.
increase pressure and temperature of air, Domestic SIBs; by central Bank of the
thus air gets compressed and warmed. country.

Answers: [3]
• The indicators to be used to assess domestic call money market, certificate of deposit,
systemic importance of the banks are as commercial bills, commercial papers,
follows: mutual funds, repo and reverse repo rate
and cash management bill.
– Size- banks having a size beyond 2% of GDP
is selected in the sample. 26. (d)

– Interconnectedness- in terms of borrowing Exp: Role of the Cabinet:


and lending from other banks.
• It is the highest decision making authority
– International links. in the politico-administrative system of a
state.
– Complexity- in terms of cost and time taken
to resolve the issues. • It is the chief policy formulating body of
the state government.
22. (b)
• It is the supreme executive authority of the
Exp: It is a fund proposed by Ratan P Watal state government.
committee on Digital Payment, from

E
savings generated from cashless • It is the chief coordinator of state
transactions to expand digital payments in administration.
OR
the country. The main suggestion made by
• It is an advisory body to the governor.
Committee of chief ministers on digital
payments headed by N Chandarbabu • It is the chief crisis manager and thus deals
Naidu is bringing back BCTT (Banking with all emergency situations.
Cash Transaction Tax) in order to promote
• It deals with all major legislative and
SC

digital payments in the country. Public


Sector Asset Rehabilitation Agency (PARA) financial matters.
is an agency suggested by Economic Survey • It exercises control over higher
to deal with twin sheet problems in appointments like constitutional
Economy. The new name of Department authorities and senior secretariat
of Disinvestment in Finance Ministry is administrators.
GS

DIPAM (Department of Investment and


Public Asset Management). 27. (d)

23. (c) Exp: CHIEF MINISTERS WHO BECAME


PRIME MINISTERS:
Exp: Tughlaqabad was founded by Giyassuddin
Tughlaq. Fatehpur Sikri was founded by • N. Modi: Chief Minister of Gujarat became
Akbar. Prime Minister in 2014.
24. (a) • V.P. Singh: from U.P., became Prime
Minister in the short lived National Front
Exp: In Kheda Satyagraha Gandhji made
government (December 1989-November
Vallabhbhai Patel as the mass leader to
carry on the struggle against the injustice. 1990).

25. (d) • P.V. Narsimha Rao: The first Prime


Minister from South India, who held the
Exp: Today there are 8 instruments or post from 1991-1996, was Chief Minister
components of Indian money market of Andhra Pradesh between 1971-1973.
especially designed to fulfill the short term
fund requirements of the different • H.D. Deve Gowda: Chief Minister of
categories of the individuals, institutions or Karnataka when he was chosen to lead the
the firms and the companies: treasury bills, United Front government in June 1996.

[ 4 ] Answers:
• Morarji Desai: Chief Minister of the 32. (c)
erstwhile Bombay State during 1952-56,
Exp: Biopiracy refers to the use of bio-resources
became the first non-Congress Prime
by multinational companies and other
Minister in March 1977.
organization without proper authorization
• Charan Singh: The Chief Minister of the from the countries and concerned people
undivided Uttar Pradesh in 1967-1968 and without giving proper compensation. The
again in 1970. financially rich countries poor in
biodiversity and indigenous knowledge
28. (a)
exploit the underdeveloped nations rich in
Exp: INSV TRAINI is the sailboat constructed at biodiversity and indigenous knowledge
Aquarius Shipyard located in Goa. It is fitted related to bio-resources. The exploitation is
with advanced features such as satellite done to develop modern commercial
communications and Raymarine navigation applications and also to save time,
suite. It is used for unassisted, non-stop expenditure and energy during their
circumnavigation. It will be mode of

E
commercialization.
transport for first Indian all-woman
circumnavigation of the globe. 33. (d)

29. (c)

Exp: It was set up under Water Act, 1974.


OR Exp: Cultural (25)
1. Agra Fort (1983)

30. (d) 2. Ajanta Caves (1983)


SC
3. Buddhist Monuments at Sanchi (1989)
Exp: Decrease in temperature will not cause any
harm to the lake. 4. Champaner-Pavagadh Archaeological Park
(2004)
31. (d)
5. Chhatrapati Shivaji Terminus (formerly
Exp: The Citizen's Charter does not by itself
Victoria Terminus) (2004)
GS

create new legal rights, but it surely helps


in enforcing existing rights. It is not 6. Churches and Convents of Goa (1986)
justiciable in nature.
7. Elephanta Caves (1987)
Department of Administrative reforms and 8. Ellora Caves (1983)
Public Grievances is responsible for
coordinating, formulating and 9. Fatehpur Sikri (1986)
operationalising Citizens charters. 10. Great Living Chola Temples (1987)
It also contains penal provisions in the 11. Group of Monuments at Hampi (1986)
event of failure of service delivery.
12. Group of Monuments at Mahabalipuram
It applies to: (1984)
• Constitutional bodies. 13. Group of Monuments at Pattadakal (1987)
• Statutory authorities. 14. Hill Forts of Rajasthan (2013)
• Public-private partnerships. 15. Humayun's Tomb, Delhi (1993)

• NGOs substantially funded by the 16. Khajuraho Group of Monuments (1986)


government and companies that provide
17. Mahabodhi Temple Complex at Bodh Gaya
services under a statutory obligation. (2002)

Answers: [5]
18. Mountain Railways of India (1999) index are the major currencies being traded
today: U.S. dollar, Japanese yen, euro,
19. Qutb Minar and its Monuments, Delhi
etc.This is also the value that an individual
(1993)
consumer will pay for an imported good at
20. Rani-ki-Vav (the Queen's Stepwell) at the consumer level. This price will include
Patan, Gujarat (2014) any tariffs and transactions costs associated
with importing the good.
21. Red Fort Complex (2007)
36. (c)
22. Rock Shelters of Bhimbetka (2003)
Exp: It is voluntary in nature, but the states can
23. Sun Temple, Konârak (1984) incorporate them in their building bylaws.
24. Taj Mahal (1983) The Building Code is used by local bodies
for framing building bylaws. Builders have
25. The Jantar Mantar, Jaipur (2010) to give a certificate mentioning that the
building has been constructed as per the
Natural (7)
planned design submitted to the local

E
1. Great Himalayan National Park bodies.
Conservation Area (2014)
37. (a)
OR
2. Kaziranga National Park (1985)
Exp: Under HRIDAY launched in January, 2015,
3. Keoladeo National Park (1985) heritage related infrastructure development
is being taken up in 12 identified cities at a
4. Manas Wildlife Sanctuary (1985) total cost of Rs.500 cr. Central government
SC

5. Nanda Devi and Valley of Flowers National will meet the entire expenditure under the
Parks (1988) scheme. But, the states and local urban
bodies are requested to supplement their
6. Sundarbans National Park (1987) resources for rapid development of heritage
Western Ghats (2012) cities. The development initiatives covered
it includes improvement of water supply,
34. (d) sanitation, drainage, waste management,
GS

footpaths, approach roads, street lights,


Exp: Deforestation will help in bringing more
electricity wiring, tourist conveniences,
land into cultivation and thus will increase
landscaping and such citizen services. The
the agricultural expansion.
12 cities selected for the scheme are Ajmer,
35. (c) Amritsar, Amravati, Badami, Dwarka,
Gaya, Warangal, Puri, Kanchipuram,
Exp: The real effective exchange rate (REER) is Mathura, Varanasi and Velankanni.
the weighted average of a country's
currency relative to an index or basket of 38. (b)
other major currencies adjusted for the
Exp: Granite, gabbro, pegmatite, basalt, volcanic
effects of inflation. The weights are
breccia and tuff are some of the examples
determined by comparing the relative trade
of igneous rocks.
balances, in terms of one country's
currency, with each other country within Depending upon the mode of formation,
the index. This exchange rate is used to sedimentary rocks are classified into three
determine an individual country's currency major groups: (i) mechanically formed -
value relative to the other major currencies sandstone, conglomerate, limestone, shale,
in the index, as adjusted for the effects of loess etc. are examples; (ii) rganically
inflation. All currencies within the said formed- geyserite, chalk, limestone, coal etc.

[ 6 ] Answers:
are some examples; (iii) chemically formed 43. (b)
- chert, limestone, halite, potash etc. are
Exp: Corbett National Park is situated in the
some examples.
foothills of the Sub- Himalayan belt in
Gneissoid, syenite, slate, schist, marble, Nainital districts of Uttarakhand state in
quartzite etc. are some examples of India. Established in the year 1936 as Hailey
metamorphic rocks. National Park, Corbett has the glory of
being India's oldest and most prestigious
39. (c) National Park. It is also being honoured as
Exp: Rise in sea level is associated with positive the place where Project Tiger was first
river rejuvenation. launched in 1973. Corbett National Park
covers an area of 521 sq. km and together
40. (c) with the neighboring Sonanadi Wildlife
Sanctuary and Reserve Forest areas, forms
41. (d)
the Corbett Tiger Reserve. Corbett is one of
Exp: Inflation tax is not an actual legal tax paid the richest bird regions of the Country and

E
to a government; instead "inflation tax" has been declared as an 'Important Bird
refers to the penalty for holding cash at a Area' (IBA) by Birdlife International.
time of high inflation. When the
government prints more money or reduces
OR
interest rates, it floods the market with cash,
which raises inflation in the long run. If an
44. (a)
Exp: statement 1 is correct whereas statement 2
is incorrect as new series will bring better
investor is holding securities, real estate or sync between IIP and GDP data.
other assets, the effect of inflation may be
SC
negligible. If a person is holding cash, At present, we can't really compare WPI
though, this cash is worth less after and CPI. This is not only because their
inflation has risen. The degree of decrease composition of goods is different but more
in the value of cash is termed the inflation importantly their base years are very much
tax for the way it punishes people who hold different. Similarly, when one looks at the
assets in cash, which tend to be lower- and IIP and GDP numbers, the IIP numbers are
middle-class wage earners. showing negative growth for the
GS

manufacturing sector in 2016-2017 but the


42. (d) GDP numbers are showing 7.5% growth
Exp: RKSK programme was launched by the in manufacturing. The new series to be
Union Ministry of Health and Family released by end-April will have a new base
Welfare in January 2014 with an aim to year of 2011-12-like it is for the Gross
cater and address health and development Domestic Product-for the Index of Industrial
needs of the country's adolescents. To guide Production (IIP) and Wholesale Price Index
the implementation of this programme, the (WPI). Not only will the new series improve
ministry in collaboration with UNFPA has the accuracy of the data, it will also make
developed a National Adolescent Health it comparable to GDP; in addition by
Strategy. The strategy focuses on age groups bringing up the base year the data is a better
10-14 years and 15-19 years with universal reflection of the contemporary economy
coverage. It covers males and females in including industrial production.
school and out of school, in urban and rural,
married and unmarried and vulnerable and 45. (a)
under-served. The Resource Kit has been Exp: Certiorari:
developed by United Nations Population
Fund (UNFPA) and Population Foundation In the literal sense, it means 'to be certified'
of India (PFI). The kit comprises i) Activity or 'to be informed'. It is issued by a higher
Book, ii) Bhranti-Kranti Game iii) Question- court to a lower court or tribunal either to
Answer Book and iv) Peer Educator Diary. transfer a case pending with the latter to

Answers: [7]
itself or to squash the order of the latter in • Fundamental Rights: The authority of the
a case. Parliament is also restricted by the
incorporation of a code of justiciable
It is issued on the grounds of excess of
jurisdiction or lack of jurisdiction or error fundamental rights under Part III of the
of law. Thus, unlike prohibition, which is Constitution. Article 13 prohibits the State
only preventive, certiorari is both from making a law that either takes away
preventive as well as curative. totally or abrogates in part a fundamental
right. Hence, a Parliamentary law that
Till recently, the writ of certiorari could be contravenes the fundamental rights shall
issued only against judicial and quasi- be void.
judicial authorities and not against
administrative authorities. However, in 47. (b)
1991, the Supreme Court ruled that the
certiorari can be issued even against Exp: Youth Parliament:
administrative authorities affecting rights The scheme of Youth Parliament was
of individuals. started on the recommendation of the

E
Like prohibition, certiorari is also not Fourth All India Whips Conference.
available against legislative bodies and
Its objectives are:
private individuals or bodies.
OR
• To acquaint the younger generations with
46. (b)
practices and procedures of Parliament;
Exp: The Indian Parliament cannot be regarded
as a sovereign body as there are 'legal' • To imbibe the spirit of discipline and
restrictions on its authority and jurisdiction. tolerance cultivating character in the minds
SC

The factors that limit the sovereignty of of youth; and


Indian Parliament are: • To inculcate in the student community the
• Written Nature of the Constitution: The basic values of democracy and to enable
Constitution is the fundamental law of the them to acquire a proper perspective on the
land in our country. It has defined the functioning of democratic institutions.
authority and jurisdiction of all the three
GS

The ministry of parliamentary affairs


organs of the Union government and the
nature of interrelationship between them. provides necessary training and
Hence, the Parliament has to operate within encouragement to the states in introducing
the limits prescribed by the Constitution. the scheme.

• Federal System of Government: India has a 48. (a)


federal system of government with a
Exp: Option 1 is correct. Option 2 is the objective
constitutional division of powers between
of Marginal standing facility. Option 3 is
the Union and the states. Both have to
the objective of SLR.
operate within the spheres allotted to them.
• System of Judicial Review: The adoption of 49. (c)
an independent Judiciary with the power Exp: Features of the Act of 1861:
of judicial review also restricts the
supremacy of our Parliament. Both the It initiated the process of decentralisation
Supreme Court and high courts can declare by restoring the legislative powers to the
the laws enacted by the Parliament as void Bombay and Madras Presidencies.
and ultra vires (unconstitutional), if they
contravene any provision of the It empowered the Viceroy to make rules
Constitution. and orders for the more convenient trans-
action of business in the council.

[ 8 ] Answers:
Features of the Act of 1892 more than other colours because it travels
as shorter, smaller waves. This is why we
It increased the number of additional (non- see a blue sky most of the time.
official) members in the Central and
provincial legislative councils, but The ocean is blue because water absorbs
maintained the official majority in them. colours in the red part of the light spectrum.
Like a filter, this leaves behind colours in
It increased the functions of legislative
councils and gave them the power of the blue part of the light spectrum for us to
discussing the budget and addressing see. The ocean may also take on green, red,
questions to the executive. or other hues as light bounces off of floating
sediments and particles in the water.
50. (c)
57. (c)
51. (c)
Exp: DDT is thus banned first by the US
Exp: Telemarketing services is an example of BPO
Government. A worldwide ban on
whereas business research is an example

E
of KPO. agricultural use was formalized under the
Stockholm Convention on Persistent
52. (d)
Exp: They clashed at three different regions such
as Marathwada country, Krishna -
OR 58.
Organic Pollutants.

(d)

Godavari Delta and Tungabhadra Doab. Exp: All are true. C. Rangarajan Committee was
also for poverty estimation. Bibek Debroy
53. (c)
SC
committee was for restructuring Indian
Exp: Agni V is an intercontinental ballistic Railways.
missile with a range of 5,500 to 5,800 km.
It is nuclear capable, with a payload 59. (b)
capacity of 1,500 kg of high-explosive Exp: The National Calendar based on the Saka
warhead. Canister-launch system of Agni-
Era, with Chaitra as its first month and a
GS

V missile will give the forces the requisite


normal year of 365 days was adopted from
operational flexibility. With this system,
ballistic missile can be transported and can March 22, 1957 along with the Gregorian
be launched from any place. calendar for the following official purposes:
(i) Gazette of India, (ii) news broadcast by
54. (b) All India Radio, (iii) calendars issued by the
Exp: Fohn occurs in Europe and Chinook occurs Government of India and (iv) Government
in America are dry winds experience on communications addressed to the public.
the leeward side of mountain air descends
Dates of the National Calendar have a
this very helpful for agriculture at times of
frost`. Chinook is also called snow eater. It permanent correspondence with dates of
is known to increase temperature extremely the Gregorian calendar, 1 Chaitra falling
quickly 35 F in 15mins. on March 22 normally and on March 21 in
leap year.
55. (d)
60. (c)
56. (b)
Exp: Epeirogenic processes involving uplift or
Exp: Sunlight reaches Earth's atmosphere and
warping of large parts of the earth's crust;
is scattered in all directions by all the gases
and particles in the air. Blue light is scattered Orogeny is a mountain building process
in all directions by the tiny molecules of air whereas epeirogeny is continental building
in Earth's atmosphere. Blue is scattered process.

Answers: [9]
• WPI weights primarily based on national
accounts and enterprise survey data and
CPI weights are derived from consumer
expenditure survey data.
65. (d)
Exp: They did not help in transmitting anything
61. (c) because they themselves did not get effected
by Renaissance. They did not change trade
Exp: Shipkila : Himachal Pradesh patter , only monopolised certain items for
few decades only.
62. (a)
66. (a)
Exp:
Exp: The type of entities which are called shadow
banks elsewhere are known as the non-
banking finance companies (NBFCs) in
India. NBFCs in India can provide lending

E
facility to customers but cannot take
deposits from banks. They are subject to
63. (a)
OR
little regulation from the market, such as
Exp: After several European Union states ratified maintaining CRR and SLR, and hence they
the agreement in October 2016, there were cannot access to RBI for funding.
enough countries that had ratified the 67. (a)
agreement that produce enough of the
SC

world's greenhouse gases for the agreement Exp: Cutting the garden into four squares was
to enter into force. The agreement went into generally adopted by Mughals.
effect on 4 November 2016.
68. (d)
64. (a)
Exp: The Act also requires every public authority
Exp: Key differences between WPI & CPI to computerize their records for wide
dissemination and to proactively certain
GS

• Primary use of WPI is to have inflationary categories of information.


trend in the economy as a whole. However,
CPI is used for adjusting income and The Act covers the whole of India except
expenditure streams for changes in the cost Jammu and Kashmir, where J&K Right to
of living. Information Act is in force.

• WPI is based on wholesale prices for Centre and state information commissions
primary articles, administered prices for are responsible for governing RTI and
fuel items and ex-factory prices for Central Information Commission has no
manufactured products. On the other jurisdiction over it.
hand, CPI is based on retail prices, which A person from disadvanced community
include all distribution costs and taxes. who desires to seek some information from
• Prices for WPI are collected on voluntary public authority does not require to pay the
basis while price data for CPI are collected fees.
by investigators by visiting markets. 69. (b)
• CPI covers only consumer goods and Exp: Rajya Sabha Chairman has recognised a
consumer services while WPI covers all group of 22 MPs belonging to smaller parties
goods including intermediate goods and certain Independents as a consolidated
transacted in the economy. bloc - the United Group.

[10] Answers:
This is only the third time in the history of development, maintenance and operation
Indian Parliament consolidated bloc has of the Dedicated Freight Corridors. Eastern
been recognised, the first was in 1983 and Dedicated Freight Corridor also covers the
the second in 1990. states of Jharkhand and West Bengal.
The newly recognised bloc will find a place 73. (c)
in the Business Advisory Committee (BAC)
that decides time allotment. Thus, MPs of Exp: Total internal reflection is the phenomenon
bloc will secure more time to speak in House which occurs when a propagated wave
debates. At present, time allotted to parties strikes a medium boundary at an angle
to speak on debates depends entirely on larger than a particular critical angle with
their strength in the House. Earlier due to respect to the normal to the surface. If the
their solitary or numerically lean status, refractive index is lower on the other side
members of this bloc had as little as three of the boundary and the incident angle is
minutes of speech time. greater than the critical angle, the wave
cannot pass through and is entirely

E
A grouping of this kind will make it possible
reflected. The critical angle is the angle of
for a party like Sikkim Democratic Front
incidence above which the total internal
with a single MP to speak for as much time

70.
as say, a Samajwadi Party with 19 MPs in
Rajya Sabha.
(a)
OR reflection occurs. This is particularly
common as an optical phenomenon, where
light waves are involved, but it occurs with
many types of waves, such as
electromagnetic waves in general or sound
Exp: Law Commission of India is an executive
waves.
SC
body established by an order of the
Government of India. Its major function is • Light cannot always pass through denser
to work for legal reform. to rarer medium.
Its membership primarily comprises legal • If the angle of incidence > critical angle, it
experts, who are entrusted a mande by the leads to total reflection of light.
Government.
GS

• Examples : Optical fiber, Mirage effect


The Commission is established for a fixed
(Usually associated with hot deserts).
tenure and works as an advisory body to
the Ministry of Law and Justice. 74. (d)
71. (a) Exp: MUFA, PUFA and Omega-3 are essentially
the types of healthy fats which a human
Exp: First statement is the correct definition of
GDP deflator. It is better measure of body requires in suitable quantity for its
inflation than CPI and WPI because it healthy growth.
covers all goods and services produced in Monounsaturated Fatty Acids (MUFA) is
the economy. This statement makes the considered a form of healthy fat generally
third statement wrong. Unlike WPI and found in vegetable oils, nuts, seeds, olives
CPI the "basket" for the GDP deflator
etc. MUFA are liquid at room temperature.
changes from year to year with people's
consumption and investment patterns. Polyunsaturated Fatty Acids (PUFA) are
also other variance of health fats found in
72. (b)
vegetable oils, fish and sea foods etc.
Exp: The Dedicated Freight Corridor Omega-3 on the other hand is type of PUFA
Corporation of India Limited (DFCCIL) is and considered an essential to have fat. It
a corporation run by the Ministry of can't be metabolized by the body and
Railways (India) to undertake planning & necessarily obtained from the food.

Answers: [11]
The significance of healthy fats is in their 81. (c)
ability to keep the body healthy by their
multiple useful functions. They provide Exp: Amendment in the provisions of the
anti-oxidants such as Vitamin-E. Also, the constitution by Special Majority of
fatty acids help the body absorbing vital Parliament and Consent of States:
nutrients from the food. MUFA in Those provisions of the Constitution which
appropriate quantities can also help are related to the federal structure of the
prevent and treat diabetes, heart problems, polity can be amended by a special majority
cancer, obesity etc. Omega-3 is helpful in of the Parliament and also with the consent
proper brain growth and prevention of
of half of the state legislatures by a simple
many diseases.
majority. If one or some or all the remaining
75. (d) states take no action on the bill, it does not
matter; the moment half of the states give
Exp: Many varieties of landforms develop by the their consent, the formality is completed.
action of each of the geomorphic agents
There is no time limit within which the
depending upon especially the type and
states should give their consent to the bill.
structure i.e. folds, faults, joints, fractures,

E
hardness and softness, permeability and The following provisions can be amended
impermeability, etc. There are some other in this way:
OR
independent controls like (i) stability of sea
level; (ii) tectonic stability of landmasses; (iii) • Election of the President and its manner.
climate, which influence the evolution of
landforms. • Extent of the executive power of the Union
and the states.
76. (d)
• Supreme Court and high courts.
SC

Exp: It was basically a violent movement


concentrate in major cities of India. It was • Distribution of legislative powers between
a leaderless movement.This was launched the Union and the states.
in the aftermath of failure of Cripps
• Any of the lists in the Seventh Schedule.
Mission.
• Representation of states in Parliament.
77. (c)
GS

Exp: Higher the amount of oxidizable pollutants • Power of Parliament to amend the
higher will be the COD of the water body, Constitution and its procedure.
as more amount of oxygen is needed to Admission or establishment of new states
oxidize all the pollutants. is done by the parliament with the simple
78. (b) majority.

Exp: By 2020 India has decided to implement BS- 82. (b)


VI norms by skipping the BS-V norms.
Exp: A person cannot be a member of both
79. (d) houses of parliament at the same time. Thus
accordingly to the representation of people
Exp: Direct Sources: Deep Ocean drilling,
act 1951:
Volcanic Eruptions and Indirect Sources:
Mining, meteors, gravitation, magnetic field • If sitting member of one house is elected to
and seismic activities. the other house, his seat in the first house
80. (a) becomes vacant

Exp: Aerosols can be both natural and artificial. • If a person is elected to two seats in a house,
Artificial includes smoke release from the he should excuse his option for one,
thermal plants. otherwise both seats becomes vacant.

[12] Answers:
• If person is elected to both the houses of • It should not raise a question of privilege.
parliament and in default of intimation his
seat in Rajya Sabha becomes vacant. • It should not revive discussion on a matter
that has been discussed in the same session.
• Similarly, a person cannot be a member of
both the Parliament and the state legislature • It should not deal with any matter that is
at the same time. If a person is so elected, under adjudication by court.
his seat in Parliament becomes vacant if he • It should not raise any question that can be
does not resign his seat in the state raised on a distinct motion.
legislature within 14 days.
87. (d)
83. (a)
Exp: It enables service voters, including armed
84. (a) forces personnel, to cast their vote in
Exp: Top producing states are Odisha, elections through e-postal ballot.
Maharashtra, Madhya Pradesh and One-way electronic transmission of blank

E
Karnataka. postal ballot would considerably cut short
India has second largest Mangenese reserve the delay in receipt of the marked postal

85.
and hence does not require to import.
(c)
Exp: Canaries Current: North Atlantic Ocean,
OR 88.
ballot by the Returning Officers on or before
the date fixed for the counting of votes.
(d)

Falkland Current: South Atlantic Ocean. Exp: Onges: Andaman and Nicobar; Toda: Tamil
Nadu
SC

86. (b)
89. (c)
Exp: It is introduced in the Parliament to draw
attention of the House to a definite matter Exp: The call money market is important segment
of urgent public importance, and needs the of the money market where
support of 50 members to be admitted. As uncollateralized borrowing and lending of
it interrupts the normal business of the funds takes place on overnight basis.
GS

House, it is regarded as an extraordinary Schedule commercial banks (excluding


device. regional rural banks), cooperative banks
It involves an element of censure against operate in this market as both borrowers
the government and hence Rajya Sabha is and lenders while DEVELOPMENT Banks
not permitted to make use of this device. such as LIC, GIC, UTI, IDBI and NABARD
The discussion on an adjournment motion are allowed to operate as only lenders in
should last for not less than two hours and this market.
thirty minutes. 90. (c)
The right to move a motion for an Exp: Phosphates are necessary for the growth
adjournment of the business of the House and maintenance of animal and human
is subject to the following restrictions: bones and teeth while organophosphates
• It should raise a matter which is definite, are required for cell division involving
factual, urgent and of public production of nuclear DNA
importance. (deoxyribonucleic acid) and RNA
(ribonucleic acid).
• It should not cover more than one matter.
Phosphate minerals exist in soluble and
• It should be restricted to a specific matter insoluble forms in rocks and soil. Plants
of recent occurrence and should not be absorb inorganic phosphate salts from soil
framed in general terms. and change them into organic phosphate.

Answers: [13]
Animals obtain their phosphate by eating 94. (d)
plants. After death and decay, plants and
animals return phosphates to the soil. 95. (c)
Exp: The Indira Gandhi International Airport
91. (a)
in Delhi has become Asia-Pacific's only
Exp: Some algal blooms are very harmful which and one of the world's few airports to
release toxic substances which can lead to achieve a "carbon neutral" status.
death for many aquatic organism.
96. (c)
92. (c) Exp: It was introduced by Jahangir and
Exp: Like the Directive Principles, the popularised by Shahajahan on Taj Mahal.
fundamental duties are also non-justiciable. 97. (b)
The Constitution does not provide for their
direct enforcement by the courts. Moreover, Exp: The regions where farmers specialise in
there is not legal sanction against their vegetables only, the farming is known as
violation. However, the Parliament is free truck farming. The distance of truck farms

E
to enforce them by suitable legislation. from the market is governed by the distance
that a truck can cover overnight, hence the
Original constitution does not contained the name truck farming
OR
fundamental duties. They are incorporated
In addition to market gardening, a modern
by enacting the 42nd Constitutional
development in the industrial regions of
Amendment Act in 1976 and a new part,
Western Europe and North America is
namely, Part IVA was added in the
factory farming.
constitution.
SC

98. (c)
The Fundamental Duties are confined to
citizens only and do not extend to Exp: Ecology is the scientific analysis and study
foreigners. of interactions among organisms and their
environment. It is an interdisciplinary field
To protect and improve the environment that includes biology, geography, and Earth
and to safeguard forests and wild life is one science. Ecology includes the study of
GS

of the directive principles of the state policy interactions organisms have with each
and also the fundamental duty. other, other organisms, and with abiotic
components of their environment.
93. (c)
The scientific study of plants and animals
Exp: Blue carbon is the carbon captured by the in their relation to each other and their
world's oceans and coastal ecosystems. The relation to environment is known as
carbon captured by living organisms in ecological system.
oceans is stored in the form of biomass and
sediments from mangroves, salt marshes, A terrestrial ecosystem is an ecosystem
seagrasses and potentially algae. It is stored found only on landforms. Six primary
carbon in the plants. terrestrial ecosystems exist: tundra, taiga,
temperate deciduous forest, tropical rain
Black carbon (BC) is the most strongly light- forest, grassland and desert.
absorbing component of particulate matter
(PM), and is formed by the incomplete 99. (c)
combustion of fossil fuels, biofuels, and Exp: Both the options are correct. The primary
biomass. BC is emitted directly into the aim of LAF is to assists banks to adjust to
atmosphere in the form of fine particles their day-to-day mismatches in liquidity,
(PM2.5). via Repo and Reverse repo operations.

[14] Answers:
100. (b) Convention to Combat Desertification
(UNCCD), Stockholm Convention on
Exp: The GEF also serves as financial mechanism Persistent Organic Pollutants (POPs) and
for the following conventions: Minamata Convention on Mercury.
Convention on Biological Diversity (CBD), Geneva Convention is not related to the
United Nations Framework Convention on Environment.
Climate Change (UNFCCC), UN

E
OR
SC


GS

Answers: [15]
Paper Code: 1 A

Mock - 1 Paper - 1
Time Allowed: 2 Hrs. Maximum Marks: 200

Roll No.:

INSTRUCTIONS

1. IMMEDIATELY AFTER THE COMMENCEMENT OF THE EXAMINATION, YOU SHOULD CHECK


THAT THIS TEST BOOKLET DOES NOT HAVE ANY UNPRINTED OR TORN OR MISSING
PAGES, ETC. IF SO, GET IT REPLACED BY A COMPLETE TEST BOOKLET.

2. This Test Booklet contain 100 questions. Each question comprises four responses (answers). You will
need to select the response which you want to mark on the OMR Sheet. In any case, choose ONLY
ONE response for each question. More than one response will be treated as wrong answer.

3. You have to mark all your responses ONLY on the separate OMR Sheet provided.

4. All questions carry equal marks. (2 Marks Each)

5. For each question for which a wrong answer has been given by the candidate, one third (-0.33 %)
of the marks assigned to that question will be deducted as penalty. There will be no penalty for a
question which is left blank.

6. While writing Name and Roll No. on the top of the OMR Sheet in appropriate boxes use “ONLY
BLACK BALL POINT PEN”.

7. After you have completed filling in all your responses you should handover the OMR Sheet to the
invigilator. You are permitted to take away the Test Booklet.

DO NOT OPEN THIS TEST BOOKLET UNTIL YOU ARE ASKED TO DO SO


1. The National Bureau of Animal Genetic 4. Recently Hindu New Year was celebrated.
Resources (NBAGR) has accorded genetic Which of the following pair(s) related to it is/
recognition to the Kendrapara breed. Which are correct?
of the following statements related to the 1. Odia : Maha Sankranti
National Bureau of Animal Genetic Resources
2. Assamese : Rangali Bihu
is/are correct?
3. Kannadigas : Baisakhi
1. It conducts systematic surveys to
characterise, evaluate and catalogue farm Codes:
livestock and poultry genetic resources
(a) 1 and 2
and to establish their National Data Base.
(b) Only 2
2. It designs methodologies for ex situ
conservation and in situ management and (c) Only 3
optimal utilization of farm animal genetic (d) All
resources.
5. Consider the following statements regarding
3. It helps in inventorisation, conservation,
the seismic waves:
improvement and sustainable utilisation
of animal genetic resources of the country. 1. P waves can travel through solids while
S wave can travel though solid, liquid and
Codes: gases.
(a) 1 and 2 2. Shadow zone of P wave is wider than

E
(b) 2 and 3 shadow zone of S waves.
(c) Only 2 3. P wave vibrate in parallel while S wave
OR
vibrate perpendicular to direction of wave
(d) All
propagation.
2. Brazil's National Biosafety Committee has
Codes:
approved the releases of Genetically Modified
Mosquitoes for the control of which of the a) 1 and 2
b) 1, 2 and 3
SC

following diseases?
1. Dengue c) Only 3
2. Malaria d) None
3. Zika
6. Which of the following statement correctly
4. Swine Flu define Petrology?
(a) It is science of rocks deals with mineral
GS

Codes:
composition, texture, structure, origin
(a) Only 3
occurrence etc .
(b) 2 and 3
(b) It is science of soils deals with mineral
(c) 1 and 3 composition, texture, structure, origin
(d) 2, 3 and 4 occurrence etc.
(c) It is science that deals with Volcanoes, its
3. Which of the following is/are the
origin, location and causes.
environmental benefits of using compost as
fertilizer? (d) It is science that deals with Earthquake,
its origin, location and causes.
1. It prevents soil erosion.
2. It improves the soil nutrient composition. 7. Which of the following will cause inflation in
3. It increases the water retention capacity. the country?
4. It always leads to faster growth of crops. 1. Monetary easing by RBI
2. Supply side constraints
Codes:
3. Expansionist fiscal policy
(a) 1, 2 and 3
4. Engaging in War
(b) 3 and 4
5. Rise in imports
(c) 2 and 4
(d) All Codes:

2
(a) 1 and 3 1. It is one of the provinces of Pakistan.
(b) 2, 3 and 4 2. India considers it as part of the undivided
(c) 3 and 5 Jammu and Kashmir, while Pakistan sees
it as a separate from PoK.
(d) 1, 2, 3 and 4
3. China-Pakistan Economic Corridor
8. Consider the following statements regarding (CPEC) passes through this region.
UDAN scheme of the Ministry of Civil
Aviation: Which of the above statements is/are correct?
1. It seeks to provide connectivity to tie II (a) Only 1 (b) 1 and 2
and tier III cities through creation of new (c) 2 and 3 (d) All
air-strips and air ports.
2. A Regional Connectivity Fund (RCF) will 12. Consider the following statements regarding
be created to meet the Viability Gap Pradhan Mantri YUVA Scheme:
Funding requirements under the scheme. 1. The Scheme will be solely implemented
by Ministry of Skill Development and
Which of the above statements is/ are correct? Entrepreneurship.
(a) Only 1 2. The Scheme will provide classroom
(b) Only 2 Entrepreneurship Education to about 15
lakh students across the country through

E
(c) Both
ITIs.
(d) None

9. Considering the naval exercise with foreign


navy which of the following exercises is/are
done with UK?
1. Malabar-15
OR Which of the above statements is/are correct?
(a) Only 1
(b) Only 2
(c) Both
2. Varuna-15 (d) None
SC
3. Konkan-15 13. Considering co-operative farming which of the
4. Simbex-15 following countries are successfully practicing
it?
Select the correct answer using below codes:
1. Denmark
(a) Only 1
2. Netherland
GS

(b) 2 and 3
3. Italy
(c) Only 3
4. Russia
(d) Only 4
Codes:
10 With respect to the Hindustani music,
(a) 1 and 3
consider the following statements:
(b) 2 and 3
1. Dhrupad is related with effort from vocal
chords and lungs. (c) 1, 2 and 3
2. Dhamar is the play of Krishna during (d) All
holy.
14. Which one of the following statement is
3. Thumri is the religious devotional songs. correct related to the industrial location?
Which of the above statements is/are (a) Cheap hydroelectric energy has facilitated
incorrect? the development of textile mill industry
along the Eastern India.
(a) Only 1
(b) Sugar, cotton textiles and vegetable oils
(b) 1 and 2
are footloose industries.
(c) Only 3
(c) Industries have been set up in port towns
(d) None during Britishers due to better
connectivity.
11. Consider the following statements regarding
the Gilgit Baltistan Region: (d) All of the above

3
15. Match the following parliamentary 3. It allows for biometric identification (via
committees: Aadhaar) to access bank account.
A. Business 1. Examines 4. It offers remittances services to the
Advisory public customers.
Committee expenditure
B. Select Committee 2. Examines the Which of the above statements are correct?
budget estimates (a) 1, 2 and 3
C. Estimates 3. Considers a bill (b) 1, 2 and 4
Committee and submits a (c) 1, 3 and 4
report
(d) 2, 3 and 4
D. Public Accounts 4. Prepares time table
Committee for session 19. Consider the following statements related to
the National Green Tribunal:
Codes:
1. NGT is a constitutional body for the cases
A B C D relating to environmental protection and
(a) 1 2 3 4 conservation of forests.
(b) 2 3 4 1 2. The Tribunal is strictly bound by the
(c) 3 1 2 4 procedure laid down under the Code of
(d) 4 3 2 1 Civil Procedure, 1908.

E
3. The Tribunal is mandated to make and
16. Which of the following is the theme of world endeavour for disposal of appeals within
day of Social Justice 2017 celebrated by UN?
OR
6 months of their filing.
(a) Ending poverty, human trafficking and
forced labour. Codes:
(b) Preventing conflict and sustaining peace (a) Only 1
through decent work. (b) 2 and 3
SC

(c) A Just Transition - environmentally (c) Only 3


sustainable economies and societies. (d) All
(d) Social Justice and Decent Life for All.
20. Recently Andhra Pradesh is looking for river
17. Which of the following atmospheric linking project on Godavari and Penna. Which
phenomenon occurs in leeward side of a of the following statements is/are true about
mountain or hill during rainfall? these two rivers?
GS

1. A decrease in altitude decreases pressure. 1. Godavari is the third largest river of India
2. Decrease in altitude increase temperature after Ganga and Indus.
of air. 2. Penna rises from Nandi hills and drains
3. Air gets expanded. into the Arabian sea.
4. The relative humidity increases. 3. Koyna dam is situated on one of the
tributaries of Godavari.
Select the correct answer using the codes
below: Codes:
(a) All (a) 1 and 2
(b) Only 2 (b) Only 2
(c) 1 and 2 (c) Only 3
(d) 2, 3 and 4 (d) None
18. With reference to Micro ATM consider the 21. Which of the following statements is/are
following statements: correct regarding Systematically Important
1. It allows deposit making, withdrawal and Banks (SIBs) of India?
balance enquiry facilities. 1. SBI and PNB are the only two banks in
2. Fund transfer is not allowed through India that have been categorized as
Micro-ATMs. domestic SIBs.

4
2. SIBs are big banks of the country that are Codes:
perceived "too big to fail". (a) Only 1 (b) Only 2
3. These are critical for the uninterrupted (c) Both (d) None
availability of essential banking services
to the country's real economy even during 25. With respect to the Indian Financial Market
crisis. which among the following instruments are
designed to fulfill short term funds
Select the correct answer using the code given requirements?
below: 1. Cash Management Bill
(a) Only 1 2. Certificate of Deposit
(b) 1 and 2 3. Call Money Market
(c) 1 and 3 4. Commercial Bills
(d) 2 and 3
Codes:
22. DIPAYAN, sometimes seen in the news, is (a) 1, 2 and 3
related to: (b) 2, 3 and 4
(a) It is a fund proposed by Committee of (c) 3 and 4
chief ministers on digital payments to be
utilized to expands digital payments in (d) All

E
the country. 26. A smaller body called cabinet is the nucleus
(b) It is a fund proposed by Watal committee of the council of ministers. It is the real Centre
from savings generated from cashless
transactions to expand digital payments.
(c) It is an agency proposed in Economic
Survey to deal with twin sheets problems
OR of authority in the state government. In this
regard which of the following statements is/
are true regarding role of the cabinet?
1. It is the highest decision making authority
of Indian economy i.e. overleveraged in the politico-administrative system of a
SC
companies and NPAs with Banks. state.
(d) It is new name of Department of 2. It deals with all major legislative and
disinvestment in finance ministry. financial matters.
3. It is the chief crisis manager and thus
23. Which of the following pair(s) is/are correct? deals with all emergency situations.
City Founder 4. It supervises the implementation of its
GS

decisions by the council of ministers.


1. Tughalaqabad Mohammad-bin-
Tughlaq Codes:
2. Fatehpur Sikri Shahajahan (a) 1, 2 and 3
3. Siri Allaudin Khalji (b) 1 and 3
4. Firozabad Firoz Shah Tughlaq (c) 2, 3 and 4
(d) All
Codes:
(a) 1 and 2 27. Which of the following is/are the people who
became Prime Minister after being Chief
(b) 2, 3 and 4 Minister of their respective states?
(c) 3 and 4 1. Narendra Modi
(d) All
2. V. P. Singh
24. Which of the following statements is/are true 3. P. V. Narsimha Rao
about the initial years of political activities by 4. H. D. Deve Gowda
Gandhiji in India? 5. Morarji Desai
1. In Chamaparan Satyagraha Gandhiji led 6. Charan Singh
the movement to relieve the peasants from
the oppression of Indigo planters. Codes:
2. In Kheda Satyagraha Gandhji made J.L. (a) 1, 2, 3, 4 and 5 (b) 1, 2, 3 and 4
Nehru as the mass leader to carry on the
struggle against the injustice. (c) 1, 2 and 3 (d) All

5
28. Which of the following defence equipments 31. Which of the following statements are true
and their salient features is/are correctly about Citizen's Charters?
matched? 1. It is justiciable in nature.
1. S-400 Missile - It is capable of exo- 2. Department of Consumer affairs are
atmospheric interception of intermediate- responsible for coordinating, formulating
range ballistic missile warheads in their and operationalizing Citizens Charters.
terminal phase. 3. It also contains penal provisions in the
2. Varunastra - It is capable of targeting event of failure of service delivery.
quiet and stealthy submarines both in 4. It is not applicable to constitutional bodies.
deep and littoral waters.
3. INSV TRAINI - It is capable of Codes:
conducting anti-piracy and rescue (a) 1 and 4
operations in India's Exclusive Economic (b) 2 and 3
Zone.
(c) Only 2
Codes: (d) Only 3
(a) 1 and 2
32. Which of the following terms associated with
(b) 2 and 3 biotechnology is/are correct?
(c) 1 and 3 1. Bioprospecting is the search for biological
(d) All resources and accompanying indigenous

E
knowledge mainly for the purpose of
29. Which of the following statements is/are true commercial exploitation.
OR
about Central Pollution Control Board? 2. Bioinformatics refers to the use of bio-
1. It works under the Ministry of resources by multinational companies
Environment and was set up under without giving proper compensation.
Environment Protection Act, 1986. 3. Cytogenetics refers to the study of the
2. It has responsibilities to conduct structure, function and abnormalities of
SC

monitoring of water and air quality, and human chromosomes.


maintains monitoring data.
3. It is responsible for maintaining national Codes:
standards under a variety of (a) 1 and 2
environmental laws, in consultation with (b) 2 and 3
zonal offices, tribal, and local (c) 1 and 3
governments.
GS

(d) All
Codes: 33. Which of the following mentioned sites are
(a) 1 and 2 recognized as UNESCO World Heritage sites?
(b) Only 2 1. Valley of Flowers National Park.
(c) 2 and 3 2. Churches and Convents of Goa.
(d) None 3. Manas Wildlife Sanctuary.
4. Rani-ki-Vav in Gujarat.
30. Which of the following activities can cause
death of a lake? Codes:
1. Agricultural runoff (a) 1 and 4
2. Industrial Effluents (b) 1, 2 and 3
3. Algal Blooms (c) 1, 2 and 4
4. Decrease in temperature (d) All
Codes: 34 Which of the following is/are the effects of
(a) 1 and 2 deforestation?
(b) 2 and 3 1. Release of carbon from its sink
(c) Only 4 2. Landslide
(d) 1, 2 and 3 3. Extreme Climate

6
4. Habitat loss 2. Central and State governments will meet
5. Decline in Agricultural Production the expenditure under the scheme on
sharing basis.
Codes: 3. One Heritage site will be developed in
(a) 1 and 2 every State under the scheme.
(b) 2, 3 and 4 Select the correct answer using the codes given
(c) 4 and 5 below:
(d) 1, 2, 3 and 4 (a) Only 1
(b) 1 and 2
35. Consider the following statements related to (c) 2 and 3
the Real Effective Exchange Rate:
(d) All
1. REER is the weighted average of a
country's currency relative to an index of 38. Which of the following are correctly matched?
other major currencies adjusted for the 1. Igneous Rocks : Granite and Chert
effects of inflation. 2. Sedimentary Rocks : Chalk and sandstone
2. It is the value that an individual consumer 3. Metamorphic Rocks : Shale and Marble
will pay for an imported good at the
consumer level. Codes:

E
(a) 1 and 2
Which of the above given statements is/are
(b) Only 2
correct?
(a) Only 1
(b) Only 2
(c) Both
OR (c) 2 and 3
(d) None

39. Which of the following characteristic features


(d) None is/are associated with negative river
rejuvenation?
SC
36. Government has recently unveiled the revised 1. Rise in Sea level
Building Code, making builders liable for the 2. Entrenched meander
structure's safety. Consider the following 3. Knick point
statements regarding it: 4. River Terraces
1. Builders have to give a certificate of
planned construction as per design Codes:
GS

submitted to state authorities. (a) Only 1


2. It includes sustainability norms like solar (b) 1 and 2
energy utilisation, usage of LED, solid (c) 2, 3 and 4
waste management and rain water (d) All
harvesting.
3. It provides for universal accessibility for 40. Which of the following are the negative
senior citizens and differently abled impacts of large dams?
citizens. 1. They lead to fragmentation of rivers
making it difficult for aquatic fauna to
Which of the above statement is/are correct? migrate, especially for spawning.
(a) Only 1 2. It lead to excessive sedimentation at the
bottom of the reservoir, resulting in rockier
(b) 1 and 2
stream beds and poorer habitats for the
(c) 2 and 3 rivers aquatic life.
(d) 1, 2 and 3 3. lead to large-scale displacement of local
communities.
37. Which of the following statements is/are 4. They create inter-state water dispute.
correct regarding 'Heritage City Development
and Augmentation Yojana (HRIDAY)'? Codes:
1. It aims to preserve and rejuvenate the rich (a) 1 and 3 (b) 2 and 3
cultural heritage of the country. (c) All (d) 3 and 4

7
41. Consider the following statements regarding 44. The government will release a new series of
'Inflation Tax': indices to measure two key economic
1. The degree of decrease in the value of variables, factory output (IIP) and wholesale
cash, due to flooding of market with cash, prices (WPI). Which of the following is/are
is termed as inflation tax. likely benefit/s of this new series of data?
2. The maximum effect of inflation tax is on 1. A better comparison between CPI and
the people/institution holding securities, WPI will be done.
real estate or other assets. 2. Anomaly between GDP and IIP data will
Which of the above statements is/are correct? increase thus will help in better data
collection.
(a) Only 1
(b) Only 2 Select the correct answer using the code given
(c) Both below.
(d) None (a) Only 1
(b) Only 2
42. The Union Ministry of Health and Family
Welfare has launched SAATHIYA Resource (c) Both
Kit and SAATHIYA SALAH mobile app for (d) None
adolescents as part of the Rashtriya Kishor
Swasthya Karyakram (RKSK) programme. 45. Which of the following statements is/are true

E
Consider the following statements regarding regarding writ-certiorari?
it: 1. It is issued by a higher court to a lower
OR
1. The Resource Kit for Peer Educators court or tribunal either to transfer a case
(Saathiyas) has been developed by WHO pending with the latter to itself or to
and Population Foundation of India (PFI). squash the order of the latter in a case.
2. The ministry has developed a National 2. It can be issued only against judicial and
Adolescent Health Strategy for age group quasi-judicial authorities.
SC

15-19 years only with universal coverage. 3. Unlike Prohibition, Certiorari is both
3. It covers only those females in school and preventive as well as curative.
out of school which are unmarried.
4. It is issued only on the grounds of excess
4. 'Saathiya Salah' will charge minimal fee of jurisdiction or lack of jurisdiction.
thus act as cost-effective information
platform for the adolescents. Codes:
GS

Which of the above statements are correct? (a) 1 and 3


(a) 1 and 2 (b) 1, 2 and 3
(b) 1, 2 and 3 (c) 2, 3 and 4
(c) All (d) All
(d) None 46. Which of the following factors limits the
43. Which of the following statements related to sovereignty of Indian Parliament?
the Corbett National Park is/are correct? 1. Written Nature of the Constitution.
1. Project Tiger was first launched in this 2. Federal System of Government.
park. 3. System of Judicial Review.
2. Corbett is one of the richest bird regions 4. Fundamental Rights.
of the Country and has been declared as
5. Directive Principles of State Policy.
an 'Important Bird Area' (IBA) by Birdlife
International. Codes:
3. It is located in Himachal Pradesh. (a) 1, 2 and 5
Codes: (b) 1, 2, 3 and 4
(a) Only 1 (b) 1 and 2 (c) 2, 3 and 5
(c) None (d) All (d) All

8
47. Which of the following statements is/are true (a) 1 and 2
regarding Youth Parliament? (b) 2 and 3
1. The scheme of Youth Parliament was (c) 1 and 3
started on the recommendation of the (d) 1 and 4
Fourth All India Whips Conference.
2. It aimed to acquaint the younger 50. Which of the following steps will encourage
generations with practices and procedures and facilitate participation of common man
in governance?
of Parliament
1. RTE
3. The Ministry of Youth and Sports affairs
provides necessary training and 2. RTI
encouragement to the states in 3. G2G in e-governnace
introducing the scheme. 4. Social Audit

Codes: Codes:
(a) Only 1 (a) 1, 2 and 3
(b) 1 and 2 (b) 2, 3 and 4
(c) 2 and 3 (c) 2 and 4
(d) Only 3

E
(d) All
51. Which of the following statements clearly
48. Which of the following is the objective of 'Base
Rate' in the Indian Economy?
(a) To enhance transparency in lending rates
of banks and enabling better assessment
of transmission of monetary policy.
OR defines the difference between KPO and BPO?
1. Business Processing Outsourcing consists
of back office and front office services
whereas KPO involves the application of
training and expertise to research.
(b) To provide overnight borrowing to
SC
2. KPO focus more on data collection,
commercial banks from RBI, at the interest updating models, patent searchers and
rate 1% higher than the current repo rate. basic data mining
(c) To enable the central banks to control the 3. Telemarketing services is an example of
amount of advances the banks can create. KPO whereas business research is an
(d) To force the banks to have sufficient vault example of BPO.
GS

cash to meet their day-to-day Codes:


requirements.
(a) Only 1
49. Consider the following features of the Act of (b) 2 and 3
1861: (c) 1 and 2
1. It initiated the process of decentralisation (d) All
by restoring the legislative powers to the
Bombay and Madras Presidencies. 52. The conflict between Vijayanagar rulers and
Bahmani Sultans was regular feature. In
2. It increased the number of additional which of the following areas they clashed with
(non-official) members in the Central and each other?
provincial legislative councils, but 1. Chola Rules Areas
maintained the official majority in them.
2. Lower Ganga Plains
3. It empowered the Viceroy to make rules
3. Tungabadra Doab
and orders for the more convenient trans-
action of business in the council. 4. Kathiaward Peninsula

4. It increased the functions of legislative Codes:


councils and gave them the power of (a) 1 and 2
discussing the budget and addressing (b) 2 and 4
questions to the executive.
(c) 3 and 4
Which of the above stated features are correct? (d) Only 3

9
53. What is the advantage of Canisterised version Which of the above statements is/are correct?
of Agni V over other forms? (a) Only 1
(a) The canisterised version of Agni V has (b) 1 and 2
higher range.
(c) 2 and 3
(b) The canisterised version of Agni V is able
(d) All
to carry large conventional warhead.
(c) The canisterised version of Agni V can be 57. Why DDT as a fertilizer is not desirable for
transported and can be launched from being used in agricultural fields?
any place. (a) Its cost is too high which puts economic
(d) All of the above. burden on farmers.
54. Consider the following statements about the (b) It makes the land permanently barren
rain-shadow winds: forever.
1. Chinook occur in European Alps also (c) It enters into the food chain and causes
known as snoweater. cancer.
2. They are known to increase temperature (d) None of these.
extremely quickly.
58. Which of the following pairs is/are correctly
3. They help in increasing agriculture matched?
productivity during the frost.
1. Tarapore Committee - Capital Account

E
Which of the above statements are correct? Convertibility.
(a) 1 and 2 2. Tendulkar Committee - Poverty
OR
Estimation.
(b) 2 and 3
3. Urijit Patel Committee - Monetary Policy
(c) 1 and 3
Reforms.
(d) All
4. Kelkar Committee - Revisiting &
55. Which of the following statements is/are true Revitalizing PPP.
SC

about "BHIM" APP?


Codes:
1. It is an App for paving the digital
payments by using the Aadhaar platform. (a) 1 and 2
2. It has the provision for referral bonus (b) 3 and 4
scheme and cashback scheme to ensure (c) 2 and 4
the promotion of digital payments. (d) All
GS

3. It is built over the IMPS infrastructure and


allows instant transfer of money between 59. Consider the following statements regarding
any two parties' bank accounts. the national calendar:
1. National calendar is based on Saka era.
Codes: 2. It was adopted from March 22, 1957.
(a) 1 and 2 3. National Calendar is started with January
(b) Only 2 as its first month.
(c) 2 and 3
(d) All Which of the above statements are correct?
(a) 1 and 3
56. Consider the following statements: (b) 1 and 2
1. Sky looks blue because the molecules in (c) 1, 2 and 3
the air scatter blue light from the sun more
(d) None
than they scatter red light.
2. The ocean is blue because water absorbs 60. Consider the following statements regarding
colours in the red part of the light Geomorphic processes:
spectrum. 1. Epeirogenic processes involve mountain
3. Red is scattered more than other colours building through severe folding and
because it travels as shorter, smaller affecting long and narrow belts of the
waves. earth's crust.

10
2. Diastrophism covers the processes of 3. The agreement is not yet ratified by
orogeny, epeirogeny, earthquakes and enough countries to come into effect.
plate tectonics come under.
3. Denudational processes cover all exogenic Codes:
processes. (a) 1 and 2
4. Exogenic processes are powered by kinetic (b) Only 2
energy only. (c) Only 3
(d) All
Which of the above statements is/are correct?
(a) Only 1 64. Which among the following statements
(b) 1, 2 and 3 correctly describes the difference between CPI
(c) 2 and 3 and WPI?
(d) All 1. Prices for WPI are collected on voluntary
basis while price data for CPI are collected
61. Which of the following passes are correctly by investigators by visiting markets.
matched? 2. CPI covers only goods whereas WPI covers
1. Zojila : J&K both goods and services.
2. Sipkila : Uttarakhand
Codes:

E
3. Nathula : Sikkim
(a) Only 1
4. Bomdila : Arunachal Pradesh
(b) Only 2
Codes:
(a) 1 and 3
(b) 1 and 2
OR (c) Both
(d) None

65. Which of the following statements is/are true


(c) 1, 3 and 4
about the Portugese impact on Indian trade
SC
(d) All and society?
62. Which of the following is/are correct 1. They were responsible for transmitting
regarding impact of inflation on saving? science & technology developed in Europe
during Renaissance to India.
1. In short term, higher inflation leads to
increase in saving rate. 2. They entirely changed the trade patterns
of India through strict control over it.
GS

2. In long term, higher inflation depletes the


saving rate in an economy. Codes:
3. In short term, when inflation falls saving (a) Only 1
rate increases.
(b) Only 2
Select the correct answer using the code given (c) Both
below: (d) None
(a) 1 and 2
(b) 2 and 3 66. Which of the followings is/are the feature/s
of Shadow Banks?
(c) None
1. They provide credit and lending facility
(d) All
to customers.
63. Climate Change is an international issue 2. They can access for funding to RBI.
concerning the entire humanity. In this regard 3. They maintain CRR and SLR with RBI.
which of the following statements is/are true 4. They accept demand deposits such as
about the Paris Climate Agreement? Savings and Current accounts.
1. It was adopted at COP-21 under the
United Nations Framework Convention Select the correct answer using the code given
on Climate Change. below:
2. It sets out the INDCs submitted by each (a) Only 1 (b) 1 and 2
signatory countries which is to be revised (c) 2 and 3 (d) Only 4
every five years.

11
67. Which of the following is/are the special (a) 1 and 2
feature(s) of the Tughlaq Architecture? (b) 2 and 3
1. Sloping walls i.e barter. (c) 1 and 3
2. Combining arch with lintel & beam. (d) All.
3. Cutting the garden into four squares.
70. Which of the following statements is/are true
Codes: regarding Law Commission of India?
(a) 1 and 2 1. Law Commission of India is an executive
(b) Only 2 body established by an order of the
Government of India.
(c) Only 3
2. Its membership primarily comprises of the
(d) All
judges of Supreme Court and High Court.
68. Which of the following statements are true 3. The Commission is established for a fixed
regarding Right to Information Act? tenure and works as an advisory body.
1. The Act requires every public authority Codes:
to computerise their records for wide
dissemination and to proactively certain (a) 1 and 3
categories of information. (b) Only 2
2. The Act covers the whole of India except (c) 2 and 3

E
Jammu and Kashmir, where J&K Right (d) All
to Information Act is in force.
71. Which among the following statements are
OR
3. Centre and state information commissions
correct regarding GDP deflator?
are responsible for governing RTI and
Central Information Commission has no 1. It is a ratio between GDP at current prices
jurisdiction over it. and GDP at constant prices.
4. A person from disadvantaged community 2. It is a better measure of inflation than
CPI and WPI.
SC

who desires to seek some information from


public authority does not require to pay 3. It covers only goods produced in the
the fees. economy.

Codes: Select the correct answer using the code given


(a) 1, 2 and 3 below:
(a) 1 and 2
GS

(b) 1 and 2
(c) 2, 3 and 4 (b) 2 and 3
(d) All (c) 1 and 3
(d) All
69. Recently 22 MPs from small parties,
independents in Rajya Sabha recognized as a 72. Which of the following statements is/are true
consolidated block (United Group). In this about the Dedicated Freight Corridors in
regard which of the following statements are India?
true? 1. These corridors are the responsibility of a
1. This is only for the first time in the history Special Purpose Vehicle under the
of Indian Parliament consolidated bloc has Ministry of Railways.
been recognized. 2. Western Dedicated Freight Corridor runs
2. The newly recognized bloc will find a from Dadri in UP to JLN Port, Mumbai.
place in the Business Advisory Committee 3. Eastern Dedicated Freight Corridor covers
(BAC) that decides time allotment. the states of Punjab, Haryana, U.P and
3. A grouping of this kind will make it Bihar only.
possible for a even with single MP to Codes:
speak for as much time as other parties.
(a) Only 1 (b) 1 and 2
Codes: (c) Only 3 (d) All

12
73. Which of the following statements is/are Codes:
correct related to the light? (a) 1 and 2
1. Light travels faster in air and slow in (b) Only 2
water. (c) Only 3
2. Total Internal reflection occurs when the (d) Only 4
angle of incidence is greater than the
critical angle. 77. Which of the following statements is/are true
about the Chemical Oxygen Demand(COD)?
Select the correct answer using below given
1. A COD test can determine the pollution
codes:
level in a water body.
(a) Only 1
2. A water body containing high levels of
(b) Only 2 oxidizable chemicals will show low COD.
(c) Both 3. It is expressed in SI units i.e milligrams
(d) None per litre(mg/L).

74. Considering the significance of healthy fats Codes:


which of the following are required for (a) Only 1
healthy growth? (b) 2 and 3

E
1. Monounsaturated Fatty Acids (MUFA) (c) 1 and 3
2. Polyunsaturated Fatty Acids (PUFA) (d) All
3. Omega-3

Select the correct answer using below given


codes:
(a) Only 1
OR
78. Which of the following statements is/are true
about BS norms?
1. BS norms in India are in accordance with
the Euro norms of Europe.
(b) 2 and 3 2. By 2020, India will implement BS-V norms
SC
(c) Only 3 all over the country.
(d) All 3. It aims to regulate the output of air
pollutants from internal combustion
75. Which of the following factors control the engines and Spark-ignition engines
evolution of landform apart from geomorphic equipment.
process?
GS

Codes:
1. Stability of sea level
(a) Only 1
2. Tectonic stability of landmass
(b) 1 and 3
3. Type and structure of landforms.
(c) Only 2
4. Climate
(d) All
Codes:
79. Which of the following sources can be used
(a) 1 and 3 to infer about earth's interior?
(b) 2 and 3 1. Meteors
(c) 3 and 4 2. Magnetic field
(d) All 3. Volcanic eruptions
4. Deep Ocean drilling
76. Which of the following statements is/are true
about Quit India Movement? 5. Gravitation field
1. It was purely a non-violent movement 6. Seismic activity
spread across the urban and rural areas.
Codes:
2. It was led by Gandhiji and J L Nehru
(a) 1, 3 and 4
from front.
(b) 1, 2 and 5
3. This was launched in the aftermath of
failure of Cabinet Mission. (c) 4 and 6
4. Gandhiji gave a call for 'Do or Die'. (d) All

13
80. Aerosols are said to have a large impact on 4. If person is elected to both the houses of
our environment. Which of the following is/ parliament and in default of intimation
are true about aerosols? his seat in Rajya Sabha becomes vacant.
1. By scattering the sunlight back into space,
it decreases the atmospheric temperature. Codes:
(a) 1, 2 and 3
2. It facilitates the deposition of pollutants
to the surface of the earth as well as to (b) 1, 2 and 4
water bodies which is damaging to the (c) 2, 3 and 4
environment. (d) All
3. Aerosols release is always natural in the
environment. 83. Consider the following statements with
reference to the cyclones:
Codes: 1. Extra-tropical cyclones have cold air at
(a) 1 and 2 their core.
(b) 2 and 3 2. The tropical cyclones derive their energy
(c) Only 3 from latent heat of water.
(d) All 3. Both are formed at low pressure centres
only.
81. Which of the following provisions of the

E
Constitution can be amended by a special Which of the following statements is/are
majority of the Parliament and the consent of correct?
OR
half of the state legislature by a simple (a) All
majority? (b) Only 1
1. Distribution of legislative powers between (c) Only 2
the Union and the States.
(d) 1 and 3
2. Power of Parliament to amend the
SC

Constitution and its procedure. 84. Which of the following statements is/are true
3. Admission or establishment of new states. about the presence of Manganese mineral in
4. Any of the lists in the Seventh Schedule. India?
1. Odisha and Maharashtra are the largest
Codes: producer of manganese in India.
(a) 1 and 2 2. It is used for steel making, paints and
GS

(b) Only 2 insecticides.


(c) 1, 2 and 4 3. Manganese imports cover 80% of India's
(d) All domestic requirements.

82. A person cannot be a member of both houses Codes:


of Parliament at the same time. Thus (a) 1 and 2
accordingly to the representation of people (b) Only 2
act 1951, which of the following statements
(c) 2 and 3
is/are true regarding vacating of seats?
(d) All
1. If sitting member of one house is elected
to the other house, his seat in the first 85. Which of the following Oceanic Currents are
house becomes vacant. correctly matched?
2. If a person is elected to two seats in a 1. Canaries Current : South Atlantic Ocean
house, he should excuse his option for
2. Falkland Current : South Pacific Ocean
one, otherwise both seats becomes vacant.
3. Agulhas Current : Indian Ocean
3. If a person is elected to seat of parliament
and state legislature at the same time then 4. Labrador Current : North Atlantic Ocean
his seat in state legislature becomes
Select the correct code:
vacant.

14
(a) 1 and 4 89. In the context of call money market, which of
(b) 2 and 3 the following statements is/are correct?
(c) 3 and 4 1. This is Inter-bank money market where
(d) 2 and 4 funds are borrowed and lent on fortnight
basis.
86. Which of the following statements is/are true
regarding Adjournment Motion? 2. Only schedule commercial banks are
allowed to operate in this market.
1. It interrupts the normal business of house.
2. It involves an element of censure against 3. No collateral is required to borrow from
the government and hence Lok Sabha and this market.
Rajya Sabha both are permitted to make
use of this device. Select the correct answer using the code given
3. If it is passed in Lok Sabha, the Council below.
of Minister need not resign from office. (a) Only 1
4. The discussion on an Adjournment (b) 1 and 2
Motion should last for not less than two
(c) 1 and 3
hours and thirty minutes.
(d) All

E
Codes:
(a) 1 and 4 90. Consider the following statements about the
mineral Phosphate:
(b) 1, 3 and 4
(c) 1, 2 and 3
(d) All OR 1. Plants absorb organic phosphate salts
from soil.
2. Animals obtain their phosphate by eating
87 Which of the following statements is/are true plants.
regarding E-Postal ballot system for services?
SC
3. Organophosphates are required for cell
1. It enables the service voters except armed
division involving production of nuclear
forces personnel, to cast their vote in
elections through e-postal ballot. DNA & RNA.
2. This two way electronic transmission of
Which of the above statements is/are correct?
blank postal ballot would considerably cut
short the delay in receipt of the marked (a) Only 2
GS

postal ballot by the Returning Officers. (b) 1 and 3

Codes: (c) 2 and 3


(a) Only 1 (d) All
(b) Only 2
91. Which of the following statements is/are true
(c) Both
about Algal Bloom in water bodies?
(d) None
1. Nutrient enrichment and rise in
88. Which of the tribes are correctly matched for temperature can cause algal bloom.
the states they belong? 2. Algal bloom is beneficial for the aquatic
1. Onges: Lakshadweep species as it increases food resource but
2. Bhotia: J&K decreases the quality of water for
3. Saharia: Rajasthan irrigation.
4. Toda: Andhra Pradesh
Codes:
Select the correct code: (a) Only 1
(a) 1 and 2 (b) Only 2
(b) 1, 2 and 3
(c) Both
(c) 2, 3 and 4
(d) None
(d) 2 and 3

15
92. Which of the following statements is/are true 2. Green buildings, solar plants and
regarding Fundamental Duties? rainwater harvesting helps in achieving
1. Like the Directive Principles, the carbon neutral status.
fundamental duties are also non-
Codes:
justiciable in nature.
(a) Only 1
2. Fundamental Duties are applicable to all
persons whether citizen or foreigner. (b) Only 2
3. To protect and improve the environment (c) Both
and to safeguard forests and wildlife is (d) None
one of the fundamental duty.
96. Which of the following statement is correct
Codes: related to the Pietra Dura?
(a) Only 2 (a) These are the running water channels in
the Mughal gardens.
(b) 2 and 3
(b) These are the half dome style of Mughal
(c) 1 and 3
architecture.
(d) All
(c) These are the floral designs made of semi-
93. With reference to the blue carbon, consider precious stones on walls.
the following statements: (d) These are the four pillars at four corners

E
1. It is stored carbon in the plants. style used in Mughal architectures.
2. It is captured by the world's oceans and 97. The regions where farmers specialise in
OR
coastal ecosystems. vegetables only, and the distance of farms
3. It is the most strongly light-absorbing from the market is governed by the distance
component of particulate matter. that a vehicle can cover overnight, named as
which of the following?
Which of the following statements is/are (a) Floriculture
SC

correct?
(b) Truck farming
(a) Only 1
(c) Factory farming
(b) 2 and 3
(d) Mixed farming
(c) Only 2
(d) 1, 2 and 3 98. Consider the following statements with respect
to the term "ecosystem":
GS

94. Various landforms are created by erosion and 1. The scientific study of plants and animals
deposition. Which of the following landforms and their relation to each other and their
is/are related to glacier? relation to environment is known as
1. Arete ecological system.
2. Cirque 2. Biosphere is the part of earth terrestrial
3. Crevasses system.

Select the correct answer using the codes Which of the above statements is/are correct?
below: (a) Only 1
(a) 1 and 2 (b) Only 2
(b) Only 2 (c) Both
(c) 2 and 3 (d) None
(d) All 99. Consider the following statements regarding
the 'Liquidity Adjustment facility':
95. Which of the following statements is/are true
about carbon neutral status? 1. The primary aim of LAF is to assists banks
to adjust to their day-to-day mismatches
1. Carbon neutrality occurs when the net in liquidity.
carbon emissions over an entire year is
2. It helps the RBI in effectively transmitting
zero.
the interest rate signals to the market.

16
Which of the above statements is/are correct? 2. Presently the GEF is the largest public
(a) Only 1 funder of projects to improve the global
environment.
(b) Only 2
3. GEF serves as financial mechanism to both
(c) Both
UNFCCC and Geneva Convention.
(d) None
Codes:
100. Which of the following statements is/are true
(a) Only 1
about the Global Environmental Facility?
(b) 1 and 2
1. World Bank and Asian Development
Bank are the GEF Agencies responsible (c) Only 3
for managing GEF projects. (d) All

E
OR
SC
GS



17
Paper Code: 1 B

Mock - 1 Paper - 2
Time Allowed: 2 Hrs. Maximum Marks: 200

Roll No.:

INSTRUCTIONS

1. IMMEDIATELY AFTER THE COMMENCEMENT OF THE EXAMINATION, YOU SHOULD CHECK


THAT THIS TEST BOOKLET DOES NOT HAVE ANY UNPRINTED OR TORN OR MISSING
PAGES, ETC. IF SO, GET IT REPLACED BY A COMPLETE TEST BOOKLET.

2. This Test Booklet contain 80 questions. Each question comprises four responses (answers). You will
need to select the response which you want to mark on the OMR Sheet. In any case, choose ONLY
ONE response for each question. More than one response will be treated as wrong answer.

3. You have to mark all your responses ONLY on the separate OMR Sheet provided.

4. All questions carry equal marks. (2.5 Marks Each)

5. For each question for which a wrong answer has been given by the candidate, one third (­0.33 %)
of the marks assigned to that question will be deducted as penalty. There will be no penalty for a
question which is left blank.

6. While writing Name and Roll No. on the top of the OMR Sheet in appropriate boxes use “ONLY
BLACK BALL POINT PEN”.

7. After you have completed filling in all your responses you should handover the OMR Sheet to the
invigilator. You are permitted to take away the Test Booklet.

DO NOT OPEN THIS TEST BOOKLET UNTIL YOU ARE ASKED TO DO SO


Directions for question 1 to 3: Read the following inhibit female literacy and educational development
passage and answer the questions that follow. Your of women. Gender inequality is one of the most
answers to these questions should be based on the crucial and yet one of the most persistent disparities
passage only. in India where differences in female and male literacy
rates are glaring, more so in the rural areas and
Passage I
among the disadvantaged sections of society. The
About two­third of the illiterate adults in the world growth rate for female literacy in the last decade has
are females. As the world works to implement the been 3% higher than the growth rate for male literacy
goals of Education for All (EFA) and the Millennium resulting in a decline in the absolute numbers of
Development Goals (MDG), Women's literacy illiterate women ­ from 200.7 million in 1991 to 190
occupy a place of importance. The United Nation's million in 2001. Gender differential in education,
Literacy Decade (UNLD ­ 2003­12) provides a special however, continues to be high at 21.7%. This can be
focus for enhanced collective endeavors and is a attributed to a number of factors ­ lack of access to
chance that must not be lost. In his book schools, lack of toilets and drinking water, parents
"Development as Freedom", Amartya Sen observes feeling insecure about sending girl children, poor
repeatedly that expansion of literacy specially female quality of education in government schools, and high
literacy, has a positive effect on basic aspirations such fees charged by the private ones.
as life expectancy and political voice. It is at this level,
According to the Census of India 2001, the female
the discrimination is conceived putting a stop to her
literacy rate increased from 39.29 per cent in 1991
Education and Development.
to 54.16 in 2001 (i.e. by 14.87 percentage points):

E
There is a growing sense of momentum and whereas, in case of males it increased from 64.13
recognition globally around literacy in general and percent to 75.85 per cent (i.e. by 11.69 percentage
Women's Literacy in particular. The global points) during the same period. It means female
OR
commitment to overcome illiteracy or reducing the literacy in the last 10 years has grown at a faster
imbalance between the literates and illiterates is rate then the male literacy rate. This has resulted in
clearly endorsed by the UN Literacy decade (2003­ narrowing down of the gender gap in literacy rate
2012) and by the report on the World Social Situation from 24.84 in 1991 to 21.69 percentage points in 2001.
(2005). Governments are engaged and a lot has been
SC

done in this area. The millennium development goals 1. According to the passage, what has been the
(MDG) have added an additional spur to action. main reason that has made the quest for
Failure to address this illiteracy predicament will learning a difficult task?
impede the Human Resource Development. Literacy (a) The neglect of education during colonial
is both an Indicator and an Instrument of times combined with social distortions.
development, and its attainment is a major factor
(b) The neglect of education since
GS

behind accumulation of Human Capital. independence as this has been the area
Education in general and mass literacy in particular, where the budget allocation has always
is a key contributor to Human Resource Development been less.
and is thus basic to any programme of Social and (c) Only the social distortions that has lead
Economic progress. The Gender Achievements and to learning being a difficult task.
Prospects in Education report (GAP, 2005) quotes (d) Gender differential in education.
"Illiteracy is a catastrophe for any child, but
particularly devastating for girls. Girls who are 2. According to the passage, what are the main
denied education are more vulnerable to poverty, factors for Gender Differentials in Education?
hunger, violence, abuse, exploitation, trafficking, 1. Lack of access to schools.
HIV/AIDS and other diseases and maternal
2. Parents feeling insecure about sending girl
mortality. If they become mothers, there is a greater
children.
chance that they will bequeath literacy and poverty
to the next generation." 3. Poor quality of education in government
schools.
The neglect of education during colonial times
4. Teacher's lack of interest in teaching the
combined with social distortions had made the quest
girl child.
for learning a rather difficult task especially for those
belonging to under privileged social groups mainly Select the correct code:
SCs/STs and women. The problem of illiteracy was
(a) Only 1 (b) 1 and 2
further aggravated by social constraints, which
(c) 1, 2 and 3 (d) All
2
3. Failure to address this illiteracy predicament 3. Inclusive growth can only be achieved in
will impede the development of …? India if local, state and national
(a) Human Resource Development. governments work in perfect
(b) The millennium development goals coordination.
(MDG). 4. In order to have growth process inclusive,
(c) The Gender Achievements and Prospects public delivery system requires priority.
in Education.
Which of the statements given above are
(d) World Social Situation. correct?
Directions for question 4 to 8: Read the following (a) 2 and 3
passage and answer the questions that follow. Your
answers to these questions should be based on the (b) 2 and 4
passage only. (c) 3 and 4
Passage II (d) 1 and 2
"Public service delivery is an integral component for 5. According to the passage, good governance
achieving growth oriented governance. Citizen will require:
centricity with the aim of ensuring citizens' welfare
(a) Citizen's welfare and satisfaction.
and satisfaction is critical for any government­local,
state or national­which aims to provide good (b) Education, healthcare, urban and rural

E
governance. India has an elaborate legal framework infrastructure and services and
and institutional structures underpinned by the employment generation.
Constitution­articulated vision of a welfare state and (c) Stability, transparency, efficiency and
by implication, provide for creation of a citizen­
centric governance structure. The average citizen is
concerned less with the convolutions of governance
and politics, or the myriad structures and levels of
OR continuity in the governance system.
(d) Good public service along with inclusive
and sustainable economic development.
government departments than with obtaining rapid 6. Government is duty bound for accelerating
SC
and equitable access to government services, whether the pace of growth. This can be achieved by:
regulatory or developmental or welfare oriented,
1. Modernising the economy, ensuring
preferably at his doorstep. Therefore, stability,
deregulation, delicensing, etc.
transparency, efficiency and continuity in the
governance systems that the citizens are most 2. By ensuring privatization and making the
immediately concerned with, is so necessary. That is system opaque.
why, our priority in India must be to place the citizen 3. By making government bare minimum
GS

at the centre of a modern public administration. noticeable.


India is a developing country and there are special 4. By making government more open and
requirements of them. It is through public service accountable.
that inclusive and sustainable economic development
can be achieved. The Government is responsible both Select the correct answer from the code given
for accelerating the pace of growth and also for below:
making the growth process inclusive. The former (a) 1 and 2
calls for economic modernization and liberalisation, (b) 1 and 3
for making government less intrusive and more
(c) 2 and 3
transparent. The latter calls for greater attention
being paid to public service delivery systems, in areas (d) 1 and 4
like education, healthcare urban and rural
7. What essential message does the author wants
infrastructure and services and employment
to convey through the passage?
generation."
(a) Ensuring inclusive growth for the
4. According to the passage: satisfaction of the citizen.
1. The average person is more concerned (b) Ensuring education, healthcare, urban
about the corruption free administration and rural infrastructure.
than anything else. (c) Sustainable development with centrality
2. The Indian Constitution has provided a of the common people.
machinery which enables the goal of a (d) Public service delivery ensuring peoples'
welfare state. satisfaction.
3
8. According to the passage, the average citizen not have anybody adjacent to their cottages. There
is least interested to understand the is only an empty cottage between Mohan and Roma.
hierarchical structure and mystical shape of Chander is adjacent to both Jayanthi and Roma.
government, rather more interested to know Tanya is next to the cottage at the beginning.
the outcome and quick service delivery. Of
the following, which conveys his feelings? 10. Who has empty cottages in both sides?
(a) People are least concerned with long list (a) Roma (b) Babu
of rules and regulations in government (c) Mohan (d) Tanya
rather they require quick redressal of their
grievances. 11. Which cottages are empty?
(b) People are fed up with the formal (a) 1, 6, 8
structure and politician's role. In fact they (b) 1, 5, 8
expect the government machinery to act
(c) 4, 5, 6
fast and ensure speedy justice at their
doorstep. (d) 5, 6, 8
(c) People have nothing to do with extremely 12. Who is in the third cottage?
large structure of the government which
(a) Jayanthi (b) Chander
is complicated and difficult to
comprehend. What they require is (c) Nobody (d) Roma
enabling governance that should facilitate
13. What is the maximum number of consecutive

E
all their requirements.
cottages that are occupied?
(d) People hardly bother what the rules and
(a) 2 (b) 3
OR
acts say and who are authorities, who
have to act, they are more concerned with (c) 1 (d) 4
obtaining rapid and equitable services in
a time bound manner. 14. Four friends P, Q, R and S go to four different
movies ­ Predator, MI­2, True Lies and Tarzan
9. Two positions of a dice are shown below: in four different theatres ­ A, B, C and D. It
SC

is known that:
1. P did not go to A or B nor did he see MI­
2 or True Lies.
2. S went to C but did not watch MI­2 or
Predator.
3. Q did not go to B but watched MI­2
GS

4. R did not watch any movie starting with


letter 'T'.

Which of the following is incorrect?


(a) P went to D and did not watch Predator.
(b) Q went to A and watched MI­2.
When three (3) dots are at the top, how many
will be at the bottom? (c) R went to B and watched Predator.
(a) 1 (b) 2 (d) S went to C and watched Tarzan.
(c) 4 (d) 5 Directions for question 15 to 18: Read the following
passage and answer the questions that follow. Your
Directions for questions 10 to 13: Answer the
answers to these questions should be based on the
questions on the basis of the information given
passage only.
below:
Passage III
Six friends went on a vacation to a hill station. They
are to be accommodated in a row of nine cottages, The bourgeoisie, historically, has played the most
numbered 1 to 9 from left. Each of them is assigned revolutionary part.
only one Cottage. The bourgeoisie, wherever it has got the upper hand,
Mohan, Tanya and Roma do not want to live in a has put an end to all feudal, patriarchal, idyllic
cottage at the end of a row. Babu and Mohan must relations. It has pitilessly torn as under the motley
feudal ties that bound man to his ‘natural superiors’,
4
and has left no other nexus between man and man 16. What is the revolutionary part, which the
than naked self­interest, than callous ‘cash author refers to, which the bourgeoise has
pay­ment’. It has drowned out the most heavenly played?
ecstasies of religious fervour, of chivalrous (a) It has transferred the social powers from
enthusiasm, of philistine sentimentalism, in the icy the serbs to the feudal class.
water of egotistical calculation. It has resolved (b) It has contributed to funding of
personal worth into exchange value, and in place of revolutions and consequent change of
the numberless indefeasible chartered freedoms, has power.
set up that single, unconscionable freedom—Free
(c) It has made monetary interest the focal
Trade. In one word, for exploitation, veiled by point of relationship between individuals
religious and political illusions, it has substituted and society.
naked, shameless, direct, brutal exploitation.
(d) It has brought radical change in the
The bourgeoisie has stripped of its halo every means of production.
occupation hitherto honoured and looked up to with
reverent awe. It has converted the physician, the 17. Which one of the following is/are correct
lawyer, the priest, the poet, the man of science, into interpretations of the views of the author?
its paid wage labourers. The bourgeoisie has torn 1. Free trade has ended the exploitation
away from the family its sentimental veil, and has earlier carried out in the garb of religious
reduced family relations into a mere money relation. and political beliefs.

E
The bourgeoisie cannot exist without constantly 2. Bourgeoise has brought a degree of
revolutionising the instruments of production, and honour to the paid wage labourers as it
thereby the relations of production, and with them
the whole relations of society. Conservation of the
old modes of production in unaltered form, was, on
the contrary, the first condition of existence for all
OR has freed them from the clutches of
feudalism and guildship, to earn money
directly for themselves.

Select the correct code:


earlier industrial classes. Constant revolutionising of
(a) Only Interpretation 1 is correct.
SC
production, uninterrupted disturbance of all social
conditions, everlasting uncertainty and agitation (b) Only Interpretation 2 is correct.
distinguish the bourgeoise epoch from all earlier (c) Both Interpretations 1 and 2 are correct.
ones. All fixed, fast frozen relations, with their train (d) Neither Interpretation 1 nor 2 is correct.
of ancient and venerable prejudices and opinions,
are swept away, all new formed ones become 18. Why did the bourgeoise bring about a change
antiquated before they can ossify. All that is solid in relations in the society?
GS

melts into air, all that is holy is profaned, and man is 1. Bourgeoise had to change the means and
at last compelled to face with sober senses his real tools of production to cater to the rising
condition of life and his relations with his kind. demand of expanding market, thereby
bringing in social changes.
The need of a constantly expanding market for its
2. Bourgeoise had to break the shackles of
products chases the bourgeoisie over the entire
guilds to wrest control over means of
surface of the globe. It must nestle everywhere, settle
production and the market. This brought
everywhere, establish connections everywhere,
change in social relations.
15. Which of the assumptions is/are correct as 3. The bourgeoise were resentful of the old
means of production, which did not have
per the passage?
space for them.
1. Bourgeoise (capitalist) causes a constant
4. The bourgeoise value only money and
change in the means and processes of profit, which ushered in a culture of value
production. of money over relationships thus changing
2. Bourgeoise has altered social relationships. the social relations.

Select the correct code: Select the correct code:


(a) Only Assumption 1 is correct. (a) 1, 2, 3 and 4
(b) Only Assumption 2 is correct. (b) 1, 2 and 4
(c) Both Assumptions 1 and 2 are correct. (c) 1 and 4
(d) Neither Assumption 1 nor 2 is correct. (d) Only 4

5
19. 4 men and 6 boys can do a piece of work in
12 days. 6 men and 15 boys can do the same 7 5 7 5
24. If x = and y = ,
work in 6 days. In how much time can 2 men 7 5 7 5
and 15 boys complete the same work? find x3 + y3.
(a) 16 days (b) 24 days (a) 1962 (b) 1692
(c) 36 days (d) None of these (c) 1269 (d) 2196

20. If m and n are integers and mn is 10. Which 25. If a person can buy at 20% less than usual
and sell at 10% more than usual his profit
of the following cannot be the value of m +
increases by 33% of the usual cost price. Find
n?
the initial profit percentage.
(a) 29 (b) 25
(a) 15%
(c) 52 (d) 50
(b) 30%
21. Two brothers Dharma and Bhim have two (c) 20%
sons each. Ram and Laxman are the sons of (d) Cannot be determined
Dharma whose ages are in the ratio 9 : 8
respectively. Bharat and Sharat are the sons 26. A 50 litres jar is filled with the finest wine.
of Bhim whose ages are in the ratio of 7 : 6. There are two measuring jars of 5 litres and
After seven years Ram is elder than Sharat by 3 litre capacities each without any calibration.
24 years. Given that the ages of Ram, Laxman, A quantity of 4 litres is to be given to a

E
Bharat and Sharat are in A.P. Find the sum customer using the jars. What is the least
of the present ages of Laxman and Bharat? number of operations required to measure
OR
(a) 32 years exactly 4 litres. An operation is said to be
complete every time an amount of wine is
(b) 48 years
transferred from one jar to another.
(c) 120 years
(a) 5 (b) 6
(d) 64 years
(c) 7 (d) 8
SC

22. A certain hostel block has ten times as many


27. Two vessels contain mixtures of water and
rooms as there are floors in it and each floor
milk in the ratio of 3 : 5 and 5 : 3. Find the
contains equal number of single occupancy
ratio in which they should be mixed to obtain
rooms. When 5 persons came to know that
a mixture that has water and milk in the ratio
10% of the rooms in the block were
of 1 : 1.
unoccupied they applied for accommodation
(a) 1 : 1 (b) 1 : 3
GS

and one of them was rejected accommodation


because the block was full after the (c) 2 : 1 (d) 1 : 2
accommodation was provided to the other
four. Find the total number of rooms in the 28. 7 coins are thrown simultaneously. What is
block. the probability of getting more number of
heads than tails?
(a) 40 (b) 50
(a) 1/3 (b) 1/4
(c) 30 (d) 20
(c) 1/2 (d) 1/7
23. Two friends Ram and Rahim started from two Directions for question 29 to 32: Read the following
places A and B towards B and A respectively passage and answer the questions that follow. Your
at 11:40 a.m. The speeds of Ram and Rahim answers to these questions should be based on the
are in the ratio 3 : 4. They meet at C (in passage only.
between A and B) spent some time together
and both started towards their destination at Passage IV
12:40 p.m. If Ram reached B at 1: 44 p.m. In democratic systems of governance, media enjoys
how much time did they spend together? a very high status because of its role as a watchdog
(a) 18 minutes over the three pillars of democracy ­ the legislature,
(b) 16 minutes the judiciary and the executive.
(c) 12 minutes The media also plays another important role, that of
(d) Cannot be determined an intermediary between the government and the
people. In this role, the media becomes the purveyor
6
of information, facilitator, exposer, aggregator of (a) Only 1 (b) Only 2
people's opinions and people's advocate. It informs (c) Both (d) None
people of the government's good work; it reports on
achievements as well as failings of the public delivery 32. According to passage what is not being done
systems; it analyses social, economic and political by media?
issues; generates and supports debate on (a) It provides an assessment of people's
government initiatives and policies; provides opinion about policies of government.
feedback on public perception of an elected
(b) It facilitates citizen group's communi­
government's performance; and so on. It also helps
non­governmental actors in the society; communicate cation with government functionaries and
with the people and the government. By doing so, with people.
the media acts as an active participant in the (c) People come to know about various good
socio­economic development of the nation. works done by government.
(d) Media helps the government in
29. Consider the following statements based on implementation of its various schemes.
the passage. Directions for question 33 to 35: Read the following
1. Media plays a passive role in the growth passage and answer the questions that follow. Your
of the people in general. answers to these questions should be based on the
2. Media does help the government by passage only.

E
informing the people about various
Passage V
achievements of government.
The word 'informal economy' is rather strongly
3. Media has become very important also
because it has become a link between the
government and citizens.

Choose the correct answer from following


OR
embedded in the economic and policy literature
today. Conceived in the course of some ILO studies
(as part of the ILO's World Employment Programme)
in Africa by Hart in 1972, it nevertheless has been
codes: severely criticized by some ­ mainly because till date
SC
(a) 1, 2 and 3 (b) 1 and 2 there is no clear definition. From the 'informal sector',
we moved to 'informal economy', and soon other
(c) 2 and 3 (d) 1 and 3
terms such as informal work and informal
30. What the author means by 'failings of the employment also emerged.
public delivery systems'. Strictly speaking each of these have different
1. Poor functioning and supervision of fair interpretations, though obviously are closely related.
GS

price shops. And depending on how we define each, statistics


2. Poor municipal services. also vary. Yet, despite all these problems and harsh
3. High level of pollution. criticism, the concept has survived. To understand
why, it may be useful to go back to the person who
4. Poor railway services.
coined the concept in the first place­Keith Hart, the
Choose the correct answer from code given economic anthropologist. According to Hart this was
below: simply a means of giving expression to 'the gap
between my experience there (in Africa) and
(a) 1 and 2 (b) 1, 2 and 4
anything that my English education had taught me
(c) 2, 3 and 4 (d) 1, 2, 3 and 4 before'.
31. What is your understanding about the term Anyone sifting through the vast literature on the
'aggregator of people's opinion'? informal economy will be surprised at the various
1. The media collects people's opinion, ways it has been described­'marginal', 'survivalist',
compiles it and brings into the notice of 'working poor', 'small capitalist sector', 'dynamic',
all concerned. 'people's economy', 'entrepreneurial' are among the
2. The media scans people's opinion and descriptions. While there are those who highlight the
selectively projects before concerned vulnerability, voicelessness and powerlessness of
authorities. those in the informal economy, some others speak of
work in the informal economy as a means to reclaim
Choose the correct answer from code given some economic power that centralized agents deny
below: them.

7
The fact that the informal economy has been used to The tourism industry is very good at spreading the
describe such varying types of economic activities ­ mantra that tourism will bring major economic
some stagnant, low productive, what can best be benefits. But how often do those benefits flow back
termed 'underemployment', and some others to the travel company, the hotel chain or the tourism
profitable and dynamic ­ only means that there exist operator who might be based overseas? What good
within this vast amorphous informal economy are thousands, even millions, of tourists to the
heterogeneous economic activities in terms of income national economy if most of what they spend goes
generating capacities, ownership of work, skills, elsewhere? The host country, or region for an
work status, productive capacity, links to other international event, must provide and maintain
sectors and to the state. airports, ports, roads and public amenities, sustain
natural and cultural heritage sites, provide water and
33. What is the thematic highlight of the passage? sewerage infrastructure for the increased traffic, but
(a) The powerlessness of the informal few international visitors rarely stay long in most of
economy the places they visit. Few in the local community can
(b) The informal economy­one word too identify what benefits are actually captured from the
many? tourist traffic.
(c) ILO studies and the informal economy The critical issue for tourism policy development in
(d) Informal employment ­ a myth? India is for the local economy to retain as much
tourism revenue as possible.
34. With the reference to the passage, consider

E
the following statements: 36. What does the author imply by the question
1. Only statistics can help us arrive at a "But how often do those benefits flow back to
the travel company, the hotel chain or the
OR
comprehensive definition of the informal
economy. tourism operator who might be based
2. The concept of an informal economy has overseas?"
been criticized for having no clear 1. It is important to understand whether the
definition. economic benefits from tourism benefit the
national economy or not.
SC

Select the correct answer using the codes given 2. It is important to understand whether the
below: tourism industry is sustainable or not.
(a) Only 1 (b) Only 2
(c) Both (d) None Select the correct answers using the codes
given below:
35. Which of the following has/have not been (a) Only 1 (b) Only 2
GS

mentioned in relation to the informal (c) Both (d) None


economy?
1. The workers in the informal economy 37. Consider the following statements:
work for centralized agents. 1. For countries hosting international events,
2. Income generating capacities are uniform people in local communities mostly do not
within the informal economy. benefit from the tourist traffic.
2. In the case of international events, the
Select the correct answer using the codes given improved infrastructure does not benefit
below: local communities in the host country.
(a) Only 1 (b) Only 2
(c) Both (d) None With reference to the passage, which of the
above statements is/are valid?
Directions for question 36 to 37: Read the following
passage and answer the questions that follow. Your (a) Only 1
answers to these questions should be based on the (b) Only 2
passage only. (c) Both
Passage VI (d) None
The real clients for any national government tourism Directions for question 38 to 41 : Read the following
policy initiatives are those who comprise the regional passage and answer the questions that follow. Your
and local communities across India. Tourists and the answers to these questions should be based on the
tourism industry are simply the actors. passage only.
8
Passage VII acrimonious debate and search for first cause. It has
When small economic crises crop up repeatedly over been suggested that China's huge savings rate may
a relatively short period of time, policymakers in each not be entirely because of domestic structural factors
country may treat each such episode as an in China but a response to the fact that the savings
independent event requiring independent action but, rate in the U S dropped sharply between 1960 and
in reality, such 'cluster crises' may be a sign of some 2010.
fundamental shift taking place in the global These adjustments give rise to economic turmoil and
economy. Hence, faced with cluster crises, it is crisis and, in addition, are politically sensitive matters
important to occasionally step back and take a more that can lead to protectionism, which can do more
holistic view of the situation. There has been some harm than good. It is important for us to recognise
research trying to do precisely that. that none of these structural shifts are caused by the
At one level, it is not difficult to see what is happening. actions of any one individual or nation. Millions of
With rapid globalisation since the end of World War little actions and thousands of scientific discoveries
II, goods and services and also capital have begun over decades and human inventiveness in general
moving much more freely across nations. In addition, have given rise to globalisation and we have inherited
and maybe even more importantly, the advance of the world we have. It is for us to take the givens as
IT has meant that it is possible for people with a given and use collective bodies such as the G­20 to
modicum of skills to sit in one country and do work ensure that we do not fall victim to protectionism. It
for another country. In brief, one of the most precious is important to remember that through all this

E
resources for economic progress, namely skilled turmoil the global pie is expanding. Hence, by having
labour, which earlier sat walled in within the effective coordinated action, it is possible to convert

OR
boundaries of their respective nations, has suddenly what appears at first as adversity into advantage.
become available to needs arising in distant parts of
the world. 38. The progress in the field of information
technology (IT) helped in the growth of world
What this has meant is that all emerging economies economy as
with a little ability to organise their workspace and
1. IT field attracted skilled manpower and
SC
impart skills to their workers are now capable of
big investment.
taking advantage of this windfall. As a consequence,
the bottom end of the skilled­labour spectrum in the 2. IT growth enabled out­sourcing of
US and Europe is now coming under competition services.
from the top end of the skilled labour band of India, 3. IT field enabled channelising of skills of
China, the Philippines, Indonesia, and several other cheaper labour of developing countries in
emerging economies. This has energised large growing enterprises of developed nation.
GS

corporations in rich and poor countries and caused


booms in various regions, like Silicon Valley in the Select the correct code:
United States. But this is also causing inequality to (a) 1, 2 and 3 (b) 2 and 3
rise in both industrialised nations and emerging (c) Only 2 (d) Only 3
economies. In a recent paper, Spence highlights how
this process is one of the causes of growth and 39. Examine whether the following assertions are
employment trends, within the US economy, correct as per what is stated in the passage.
diverging and inequality rising. And "the major 1. The growth of IT and availability of
emerging economies are becoming more competitive cheaper skilled labour has helped
in areas in which the US economy has historically economies to grow but has also led to rise
been dominant, such as the design and manufacture of inequality across the economies.
of semi­conductors, pharmaceuticals, and
2. The growth of IT and cheap skilled labour
information technology services". By the same
has also given rise to disparities in savings
argument, the skilled end of the labour markets in
rates across nations.
India and China is competing with its counterparts
in industrialised nations and, as a consequence, its Select the correct answer from the following
salaries are rising, resulting in growing inequality in options:
these countries.
(a) Only Assertion 1 is correct.
There are other domains where these kinds of inter­ (b) Only Assertion 2 is correct.
country tensions have been building up. Disparities
(c) Both Assertions l and 2 are correct.
in savings rates across nations have often led to
(d) Neither Assertion 1 nor 2 is correct.
9
40. The objective of the passage is: 43. In how many years from Year 2 has the
(a) To emphasise on the need for taking a expenditure risen by the around 15%?
holistic view of problems and issues in a (a) 1 (b) 3
cluster crises situation (c) 2 (d) Insufficient data
(b) To highlight, how growth of IT and trans­
border movement of skilled labour and 44. If in Year 6 the expenses increase at the same
services has helped in growth of economy rate as they did from Year 3 to Year 5, what
as well as created inequalities and other will be the expenditure in Year 6 (Rs. Crore )?
problems (a) 28.2 (b) 24.2
(c) To emphasise on the need for eschewing (c) 25.2 (d) 23.2
protectionism in the present day world
Directions for question 45 to 47: Read the following
economy
passage and answer the questions that follow. Your
(d) All of the above answers to these questions should be based on the
passage only.
41. Which of the following statements is/are
correct in light of what is stated in the Passage VIII
passage?
MGNREGS is playing the role of a catalyst in the
1. Cluster crises are a certain sign of some rural economy. It is playing an important role in the
fundamental shift taking place in the overall development of the rural area. MGNREGS
economy. addresses itself mainly to working people and their

E
2. Post World War II there has been much fundamental rights to live with dignity. This is one
freer movement of goods, services and of the greatest experiments undertaken by the
OR
capital across nations. Government of India to eradicate rural poverty. It
3. Due to freer movement of skilled labour, has provided protection against extreme poverty,
the bottom end of spectrum of skilled helped in economic independence of women,
labour in many developed economies is reduced crime and child labour in the rural areas.
facing competition with the bottom end The socio­ economic conditions of the rural people
SC

of spectrum of skilled labour of many has improved to a great extent due to MGNREGS.
emerging economies. The programme has immense potential to provide
better livelihood opportunities to poor rural people,
Select the correct code: focuses on poverty alleviation. A notable aspect of
(a) 1, 2 and 3 (b) 2 and 3 this programme is the participation of large number
(c) Only 2 (d) Only 3 of rural women who have sought work under
MGNREGS. The scheme is poised to progress from a
GS

Directions for questions 42 to 44: Questions are


based on the graph below: mere wage employment to sustainable development
programme. The success of MGNREGS however, will
depend on people's realization of the Act as a right.

45. Consider the following statements in respect


of above passage.
1. MGNREGS has improved the overall
social security in rural areas.
2. The programme has been able to provide
gender justice.
3. MGNREGS can fully succeed only when
people start using it as a right.
Total for all five years = Rs.100 crores.
Choose the correct statement from the code
42. The expenses of which year added to one given below:
fourth the expenses of another year equal the (a) 1 and 2
expenses of each of two other years? (b) 2 and 3
(a) 4 (b) 3 (c) 1 and 3
(c) 2 (d) 1 (d) 1, 2 and 3

10
46. MGNREGS is playing a role of catalyst in the year have to be vaccinated, the maximum
rural economy because percentage of male children born in the same
1. It has provided employment with dignity. year that can be vaccinated is:
2. It has empowered the women. (a) 50%
(b) 63%
Choose the correct option from code given
(c) 37%
below:
(d) Indeterminate
(a) Only 1 (b) Only 2
(c) Both (d) None Directions for questions 51 to 54: Answer the
questions on the basis of the information given
47. Of the following which benefit is not coming below:
out of MGNREGS? A family consists of 6 members namely A, B, C, D, E
(a) Cases of child labour has been reduced. and F. B is the brother­in­law of C's mother. C is
(b) It is a right based programme and the father of F. D is father of A and F is grandson of D.
people are fully understanding it. There are two female members in the family.
(c) It has been able to arrest rural poverty.
51. Who is the mother of C?
(d) It has provided livelihood with dignity.
(a) B
Directions for questions 48 to 50: Answer the

E
questions on the basis of the information given in (b) E
the line chart. (c) A
The following line chart gives the birth figures (in
hundreds) for a small town called Jiasarai from 1990
to 1998.
OR(d) Cannot be determined

52. Which of the following pairs represents the


female members in the family?
(a) A and E
SC
(b) B and C
(c) C and D
(d) Cannot be determined

53. The members of the family belong to how


many generations?
GS

(a) Four
(b) Two
(c) Three
48. In which year did the birth of female children
(d) Either two or three
decrease from the previous year for the first
time during the given period? 54. D has two children. One is A and who is the
(a) 1991 other one?
(b) 1993 (a) E (b) C
(c) 1994 (c) B (d) F
(d) Indeterminate
55. Two tanks of equal volume contain chemical
49. What is the average annual growth rate of A & B. The first tank contains half as much
female births from 1990 to 1998? A as B. The second tank contains one­ fourth
(a) 140% as much chemical B as A. The mixtures of the
(b) 20% two tanks are mixed in the third tank in the
ratio 3 : 5. What is the ratio of A : B finally?
(c) 15.5%
(a) 4 : 1
(d) 17.5%
(b) 5 : 3
50. If 2500 children (born in the same year) can (c) 4 : 3
be vaccinated in 1996 in Jiasarai and atleast (d) 1 : 5
50% of the female children born in the same
11
Directions for questions 56 to 58: Answer the 59. If Jatin is not sitting opposite Lamba, then
questions on the basis of the information given Francis is sitting next to whom?
below: (a) Faisal
Six friends Arun, Brian, Cory, Dorthy, Eela and Faruq (b) Eram
decided to go on a road trip on three bikes each having (c) Lamba
a rider and a passenger. If Arun is riding then neither
(d) Either Eram or Lamba
Brian nor Eela will sit with him. If Eela is riding then
Cory will sit with him. Faruq sits with Brian and 60. If Francis is not sitting next to Lamba then
Dorthy is a rider.
how many different seating arrangements are
possible?
56. If Arun is riding, then who is his passenger?
(a) Three (b) Five
(a) Faruq (b) Dorthy
(c) Four (d) Six
(c) Cory (d) Eela
61. If Hanif is sitting at one of the corner seats,
57. Who among the following can be the three
then who is sitting opposite him?
Riders?
(a) Arun, Dorthy, Eela (a) Francis (b) Daya
(b) Brian, Eela, Dorthy (c) Ajit (d) Gehna
(c) Cory, Faruq, Arun Directions for questions 62 and 63: Answer the
questions on the basis of the information given
(d) Dorthy, Cory, Arun

E
below:
58. Who among the following sits behind Dorthy Five friends Aruna, Bharti, Catherine, Drishti and
OR
if Arun is riding? Elena drive to a coffee parlor for a meeting. Each of
(a) Eela them arrived by a different car at different times.
(b) Cory Bharti arrived on a car other than Ambassador after
(c) Faruq Drishti has arrived. The person arriving on Limo car
(d) Either Eela or Cory arrived the last. Elena arrived just before the last
SC

Directions for questions 59 to 61: Answer the person and he did not drive a Mercedes car. The
questions on the basis of the information given Volkswagen owner arrived just after the Fiat driver
below: who arrived first. Three people arrived between
Catherine and Aruna.
Twelve friends Ajit, Bose, Castro, Daya, Eram,
Francis, Gehna, Hanif, Ilena, Jatin, Khosla and 62. Who arrived in Mercedes car?
Lamba are sitting around rectangular table to have
GS

dinner. There are 12 chairs around the table (a) Aruna (b) Catherine
numbered from 1 to 12 (see figure) and only one (c) Drishti (d) Bharti
person can sit on one chair (not necessarily in the
same order). Some additional information is given 63. Who arrived the third?
below: (a) Drishti
(b) Bharti
(c) Elena
(d) Cannot be determined

64. If MAJOR is coded as NCMSW then what


will be the code for ATTACK?
(a) ZUUZDL (b) BVWEHQ
(1) Francis is sitting at chair number 1, is
diagonally opposite Castro who is sitting (c) BVWEIQ (d) BVWEIP
opposite Daya.
65. In how many ways can 10 puppies (all look
(2) Eram is sitting opposite Ilena who is the alike) be sold to 6 men, given that every puppy
only person sitting between Bose and must be sold and each man must have at least
Lamba.
one puppy?
(3) Ajit is sitting opposite Khosla who is the
(a) 15 C 5 (b) 9 C 6
only person sitting between Castro and
Jatin. (c) 10 C 6 (d) 9 C 5

12
66. Six sides of a block are coloured Green, Blue, 68. Final shot means:
Red, Yellow, Orange and White in the (a) The only chance
following manner.
(b) The only chance that day
(c) The last chance that day
(d) Last chance in tournament

69. Nailed the dive means:


(a) Hurt in the dive
(b) Broken the diving board
(c) Dived successfully
(d) Dived poorly
Directions for question 70 to 72: Read the following
passage and answer the questions that follow. Your
answers to these questions should be based on the
When Blue is on the top, which colour will be
passage only.
at the bottom?
(a) Orange (b) Red Passage X

E
(c) White (d) Yellow While we were waiting for the tea to arrive, I sat
down opposite the chimpanzee and lit a cigarette.
Directions for question 67 to 69: Read the following
To my surprise, he became very excited and held out
passage and answer the questions that follow. Your
answers to these questions should be based on the
passage only.
Passage IX
OR
his hand across the table to me. Wondering what he
would do, I handed him the cigarette packet. he
opened it, took out a cigarette and put it between
his lips. He then reached out his hand again and I
After the next day's competition, I was in second gave him the matches; to my astonishment, he took
SC
place. My coach said I was "a dark horse for a one out of the box, struck it, lit his cigarette and threw
medal". the Russian and Chinese divers had been the box down on the table. Lying back in his chair,
heavily favored. At 29, I had been written off by most he blew out clouds of smoke in the most professional
people as too old to win. "Maybe you should march manner. No one had told me that Chimpanzee
before all your events," Coach joked. smoked. I wondered rather anxiously what other
But my next­to­last dive was a disaster and plunged undesirable habits he might have which his master
GS

me into fifth place. I had one final shot at a medal. had not warned me about.
As I stood on the platform ready to take my last dive, Just at that moment, the tea was brought in and the
I paused a bit longer than usual. The announcement chimpanzee greeted its appearance with loud hoots
came over the loudspeaker. "Mary Ellen Clark of the of joy. He watched me carefully while I half­filled
United States, doing a backward one­and­a­half his mug with milk and then added the tea. I had
somersault with two­and­a­half twists. been told that he had a very sweet tooth so I put in
I stepped up to the edge of the platform and turned six large spoons of sugar, an action which he greeted
my back to the water. With a quick prayer and an with grunts of satisfaction. He placed his cigarette
incredible sense of lightness, I was airborne, arcing on the table and seized the mug with both hands;
out over the pool, twisting and tumbling, the then he stuck out his lower lip very carefully and
Barcelona skyline flashing by. A micro­instant later, dipped it into the tea to make sure it was not too
I ripped into the water. I knew I had nailed the dive. hot. As it was a little warm, he sat there blowing on
When I shot back to the surface, Coach O'Brien was it vigorously until it was cool enough, and then he
yelling, "Bronze, Mary Ellen, bronze!" drank it all down without stopping once. When he
had drained the last drops, he peered into the mug
67. "dark horse for medal means" _____ to win: and scooped out all the sugar he could with his
(a) Likely forefinger. After that, he tipped the mug up on his
nose and sat with it like that for about five minutes
(b) Highly likely
until the very last of the sugar had trickled down
(c) Unlikely into his mouth. For a few minutes, he was in a world
(d) Highly unlikely of his own, far away from the threat of civilized man.

13
70. What effect did Durrell's lighting of a cigarette 73. Which of these facets of customer behavior
have on the chimpanzee? It got was not immediately realized by the bank
(a) Surprised managers of the regional bank
(b) Excited (a) Customers don’t enjoy waiting for tellers.
(c) Annoyed (b) Service is not the most important factor
in switching banks.
(d) Angry
(c) Customers only switch banks in extreme
71. The phrase "had a sweet tooth" means situations.
(a) Its tooth tasted sweet (d) Cost reduction is not a tangible benefit.
(b) Loved sweets 74. What is consumer inertia in regards to
(c) Was polite banking?
(d) Was well­mannered (a) Cost of improving service time.
(b) Cost of attracting new customers.
72. By undesirable needs author means
(c) Low motivation of consumers in
(a) Acceptable switching banks.
(b) Unacceptable (d) Increased revenues in regards to new
(c) Expected customers.
(d) Unexpected

E
75. Which of these improvements did the reduced
Directions for question 73 to 75: Read the following service time actually provide?
passage and answer the questions that follow. Your
(a) Change easy to explain to customers.
OR
answers to these questions should be based on the
passage only. (b) Helped in attracting new customers.
(c) Impossible for competitors to copy.
Passage XI
(d) Increased profit margins for the bank.
The fact that superior service can generate a
Directions for question 76 to 80: The following bar
competitive advantage for a company does not mean
SC

graph gives the monthly expenditure of a family


that every attempt at improving service will create household and the line graph that follows provides
such an advantage. Investments in service, like those the monthly medical expenses as a percentage of total
in production and distribution, must be balanced monthly expenditure. Observe these two graphs and
against other types of investments on the basis of answer the five questions that follow:
direct, tangible benefits such as cost reduction and
increased revenues. If a company is already
GS

effectively on a par with its competitors because it


provides service that avoids a damaging reputation
and keeps customers from leaving at an unacceptable
rate, then investment in higher service levels may be
wasted, since service is a deciding factor for
customers only in extreme situations. This truth was
not apparent to managers of one regional bank,
which failed to improve its competitive position
despite its investment in reducing the time a customer
had to wait for a teller. The bank managers did not
recognize the level of customer inertia in the
consumer banking industry that arises from the
inconvenience of switching banks. Nor did they
analyze their service improvement to determine
whether it would attract new customers by
producing a new standard of service that would
excite customers or by proving difficult for
competitors to copy. The only merit of the
improvement was that it could easily be described
to customers.

14
76. In how many months, the actual medical 79. If the percentage increment in the total
expenses went down as compared to the medical expenses from August to September
previous month? is same as that from July to August, what
(a) Two (b) Three percentage of total expenditure went into
medical expenses in the month of September?
(c) Four (d) Five
(a) 10%
77. What percentage of total expenditure of first (b) 17.29%
three months put together went into the (c) 22%
medical expenses?
(d) Can't be determined
(a) 15.63% (b) 15.67%
(c) 15% (d) 8.33% 80. The number of people requiring medical
attention in a month is always more than that
78. In which of the months, the actual medical in previous month. If the average monthly
expenditure increased by the largest medical expense per person is least for the
percentage as compared to the previous month of July, what is the minimum possible
month? number of members in the family?
(a) March (b) April (a) Eight (b) Ten
(c) May (d) August (c) Eleven (d) Twelve

E
OR
SC
GS

15
Paper Code: 1 B

Mock - 1 Paper - 2

E
ANSWERS

1. (a)
Exp: As stated in the Paragraph, "The neglect of
education during colonial times combined
OR the myriad structures and levels of
government departments than with
obtaining rapid and equitable access to
government services, whether regulatory or
with social distortions had made the quest
developmental or welfare oriented,
for learning a rather difficult task especially
SC
preferably at his doorstep."
for those belonging to under privileged
social groups mainly SCs/STs and women". The Constitution has stated the principle
Hence answer is (a) of welfare state and citizen in India are
more concerned with the public service
2. (c) delivery mechanism than the governance
and politics.
3. (a)
GS

Hence answer is (b)


Exp: As stated in the paragraph ­ "Governments
are engaged and a lot has been done in this 5. (a)
area. The millennium development goals
6. (d)
(MDG) have added an additional spur to
action. Failure to address this illiteracy Exp: As stated government should work for
predicament will impede the Human economic modernization and liberalization
Resource Development. Literacy is both an for making government less intrusive and
Indicator and an Instrument of develop­ more transparent and government should
ment, and its attainment is a major factor pay greater attention to enhance public
behind accumulation of Human Capital." service delivery systems to be more
Hence answer is (a) accountable and transparent. Hence
answer is (d)
4. (b)
7. (d)
Exp: As stated in the passage ­ "Constitution
8. (c)
articulated vision of a welfare state and by
implication, provide for creation of a 9. (c)
citizen­ centric governance structure. The
Sol: According to the figure, 1 will be opposite
average citizen is concerned less with the to 2, 3 will be opposite to 4 and 5 will be
convolutions of governance and politics, or opposite to 6.

Answers: [1]
Solution for questions 10 to 13:
M = 32 B
The arrangement is as follows:
1. X 3
Hence 6M + 15B = 6 × B + 15 B = 24B
2. Tanya 2

3. Jayanthi 3
Similary 2M + 15B = 2 × B + 15B = 18 B
4. Chander 2
5. Roma If 24 Boys can complete work in 6 days then
6. X 18 Boys can complete same work in 4.5
days.
7. Mohan
Hence answer is 2 men and 15 boys can
8. X complete work in 4.5 days.
9. Babu
20. (d)

E
10. (c)
Sol: (a) 25 ­ Possibility 20+5=25 and 20*5=100
OR
11. (a)
(b) 29 ­ Possibility 25+4=29 and 25*4= 100
12. (a)
(c) 52 ­ Possibility 50+2 =52 and 50*2= 100
13. (d)
(d) 50 ­ No possibilities
14. (d)
SC

21. (c)
Sol: The table is as follows:
Sol: Let Ram's age = 9x
Name Theatre Movie
Laxman's age = 8x
P D Tarzan
Bharat's age = 7y
Q A MI­2
GS

Sharat's age = 6y
R B Predator
Given 9x – 6y = 24
S C True Lies
15. (c) And as the brothers' ages form an A. P.

16. (c) 9x ­ 8x = 7y ­ 6y

Exp: As stated in the passage ­ "The bourgeoisie x = y ... (2)


has torn away from the family its Solving (1) and (2)
sentimental veil, and has reduced family
relations into a mere money relation". x=8
Hence answer is (c)
Sum of Laxman’s and Bharat’s age
17. (d) = 8x + 7y = 15x
18. (c) = 15 × 8 = 120 years
19. (d) 22. (a)
Sol: (4M + 6B) 12 = (6M + 15B) 6
Sol: As per question 5 students have applied
Solving it gives and one get rejected, this means only 4
rooms were empty. Further as per question

[ 2 ] Answers:
the emply room is 10% of the total rooms, 3rd  Empty the 3 lt. jar
thus total rooms are 40.
4th  Pour 2 ltrs. from 5 lt. Jar into 3 lt. jar
23. (c)
5th  Fill the 5 lt. jar
Sol: Let t be the time taken by Ram and Rahim
to meet at c. 6th  Pour water from 5 lt. Jar into 3 lt. Jar,

t Ram and tRahim be the time required by them Max. 1 ltrs. Can be transferred.
to travel from c their respective Now the 5 lt. Jar contains 4 ltrs. of wine.
destinations.
27. (a)
Sol: total milk = 3x + 5
Given tRam = 64 min total water = 5x + 3
(3x + 5)/(5x + 3) = 1:1

E
tRahim = tRam × Solving it give 1:1

= 36 minutes

= 48 minutes
OR 28. (c)
Sol: Total outcomes: 2^7
Favorable outcomes:
4 heads ­­> combination of HHHHTTT ­­>
SC
i.e., they travelled from 11:40 a.m. to 12:28
p.m. and they were together from 12:28 7!/(4!*3!)=35 (# of permutation of 7 letters
p.m. to 12:40 p.m. for 12 minutes. out of which 4 H's and 3 T's are identical);

24. (b) 5 heads ­­> combination of HHHHHTT ­­>


7!/(5!*2!)=21;
25. (b)
GS

Sol: Initial cost price = x 6 heads ­­> combination of HHHHHHT ­­>


7!/(6!*1!)=7;
Initial selling price = y
7 heads ­­> combination of HHHHHHH ­­
Profit = y–x > 1;
assumed C.P = 0.8 x P(H>T)=Favorable outcomes/Total
S.P = 1.1 y outcomes = (35 + 21 + 7 + 1)/2^7=1/2.

Profit2 = 1.1 y – 0.8 x 29. (c)

Given P2 = P1 + 0.33 x Exp: Media plays the active role in the growth
of people.
1.1 y – 0.8 x = y – x + 0.33 x
y = 1.3 x 30. (b)

Hence % increase = 30% 31. (a)

26. (b) Exp: Media becomes the purveyor of


information, facilitator, aggregator of
Sol: 1st Fill the 5 lt. Jar public opinion and bring that to notice of
2nd Pour its content into the 3 lt. Jar (2 lt. the government.
Remain in the 5 lt. jar)
32. (d)

Answers: [3]
33. (b) 44. (b)

Exp: As stated in the passage, till date there is Exp: The rate of increase in expenses from year
no clear definition. From the 'informal 3 to year 5 is 10%. So expenditure in year 6
sector', we moved to 'informal economy', = 22 × 1.1 = 24.2 crore.
and soon other terms such as informal work 45. (d)
and informal employment also emerged.
46. (c)
34. (b)
47. (b)
35. (d)
Solution for questions 48 to 50:
36. (a)
The number of female births in any year
37. (a) can be calculated by subtracting the
number of male births from the total
38. (b)
number of births.
39. (a)

E
40. (d)
OR
41. (c)
Exp: Due to freer movement of capital, goods and
services, the bottom end of spectrum of
skilled labour in many developed
economies is facing competition with the
SC

bottom end of spectrum of skilled labour of


many emerging economies not the
movement of skilled labour.
Solution for questions 42 to 44: 48. (b)

42. (c) 49. (d)


GS

50. (b)
1
Sol: 1st year + 3rd year = 4th year = 5th year Solution for questions 51 to 54:
4
(M) Brother (M) Husband (F)
1 B D E
17 + × 20 = 22. Hence Ans. is 2 &
4 Wife Son
Option3. Daughter (M)
Grandson

C
43. (c) A
(F)
Sol: Expenditure 2nd year = 19%
F (F)
Expenditure 4th year = 22%
51. (b)
Net Rise = 3%
52. (a)
3 53. (c)
% Rise = × 100 = 15%
19
54. (b)
Thus in 2 Year
55. (b)

[ 4 ] Answers:
Exp: Ratio of A to B in first and second mixtures 66. (b)
is 1 : 2 and 4 : 1. If we mix 3 liters of first
with 5 liters of second mixture, then ratio 67. (b)
68. (d)
of A to B finally = 69. (c)
70. (b)
= 5 : 3.
71. (b)
56. (c)
72. (b)
57. (b)
73. (b)
58. (a)
74. (c)
Solution for questions 59 to 61:
75. (a)

E
G H E J K C
Solution for questions 76 to 80:
12 11 10 9 8 7

OR January
Fabruary
Total Exp.
50000
50000
Medical Expenses
10000
6000

1 2 3 4 5 6 March 60000 9000


SC

F L I B A D April 75000 9000


59. (c) May 80000 12000
60. (b) June 65000 13000
61. (a) July 40000 10000
GS

Solution for questions 62 and 63: August 55000 11000


A Fiat 76. (a)
D Volkswagen 77. (a)
B Mercedes
25000
E Ambassador Sol: × 100 = 15.625
160000
C Limo 78. (a)
62. (d)
9000  6000
63. (b) Sol:  1
2  100  50%
6000
64. (d)
79. (d)
65. (d)
Sol: Can’t be determined as total medical
Exp: This is similar to distributing 10 similar expenses is not given.
objects to 6 receivers where each receiver
80. (d)
must receive at least one object, which is
given by n­1Cr­1 = 9C5.

Answers: [5]

You might also like